March-18-2009

  • July 2020
  • PDF

This document was uploaded by user and they confirmed that they have the permission to share it. If you are author or own the copyright of this book, please report to us by using this DMCA report form. Report DMCA


Overview

Download & View March-18-2009 as PDF for free.

More details

  • Words: 19,459
  • Pages: 90
A-PDF Merger DEMO : Purchase from www.A-PDF.com to remove the watermark

Set No. 1

Code No: R05410105

IV B.Tech I Semester Supplimentary Examinations, March 2009 ANALYSIS AND DESIGN OF STRUCTURES FOR WIND AND EARTHQUAKE EFFECTS (Civil Engineering) Time: 3 hours Max Marks: 80 Answer any FIVE Questions All Questions carry equal marks ⋆⋆⋆⋆⋆ 1. (a) Define the terms i. Epicenter ii. Hypocenter (b) Explain the various mechanisms that result in Earthquakes.

[6+10]

2. A free vibration test is conducted on an empty elevated water tank. A cable attached to the tank applies a lateral (horizontal) force of 200 kN and pulls the tank horizontally by 20 mm. The cable is suddenly cut and the resulting free vibration is recorded. At the end of 4 complete cycles, the time is 2 sec and the amplitude is 10 mm. Compute the following. (a) Damping ratio (b) Natural period of undamped vibration (c) Stiffness (d) Weight (e) Damping coefficient (f) Number of cycles required for the displacement amplitude to decrease to 2 mm. [16] 3. (a) Define Degrees of Freedom. Explain, how do you select them. (b) Formulate the Stiffness matrix for the frame shown in the figure 3.

[6+10]

Figure 3 4. (a) A R.C. framed building is 45 m × 15 m in plan and 50 m in height consisting of stories 4 m in height. It is braced in the longitudinal direction by rigid frame action and by a R.C. infill wall in the transverse direction. Determine 1 of 4

Set No. 1

Code No: R05410105

the design wind force on the framed building. Assume that the building is situated in terrain category 3 with basic wind speed of 50 m/sec in a fully developed velocity profile. (b) What are the factors that affect the response of the structure during an earthquake? [8+8] 5. (a) Is it desirable to have high strength concrete in earthquake resistant design of reinforced concrete structures ? Justify your answer. (b) Distinguish between the Response spectra and Design spectra.

[8+8]

6. A 4 storeyed (G+3) R.C. office building built in medium soil has a ground plan of 20m × 15m. The imposed load on the roof is 1.5 kN/m2 and that on the floors is 3 kN/m2 . Determine the seismic load on the frame by I.S.1893 code. Assume that there is a shear wall in the Y-direction. The roof and floor slabs are 150 mm thick. The size of the beam is 250mm × 400 mm. The size of the column is 400mm × 500mm. The total height of the building is 12 m and each floor is 3m. There is a 12cm thick brick wall around each floor of the building. The location is in Zone-III. Assume, SMRF construction is used. AS shown in figure 6. [16]

2 of 4

Set No. 1

Code No: R05410105

Figure 6 7. (a) What are the measures, one can take for improving the ductility of a reinforced concrete structure? (b) What are the design requirements of beam-column joints in earthquake resistant design? [8+8] 8. A bar bell type shear wall with central part 3600 mm x 150 mm and two 400 mm x 400 mm strong bands at each end is supported on a footing 8 m x 4 m, which rests on soil whose modulus is 30,000 kN/m3 . Determine the lateral stiffness of the 3 of 4

Set No. 1

Code No: R05410105

wall. Use M 20 grade concrete. The height of the wall is 13 m. Also calculate the lateral stiffness of the wall for a point load at the top, assuming that the wall is rigidly held at its base. Shown in figure 8. [16]

Figure 8 ⋆⋆⋆⋆⋆

4 of 4

Set No. 2

Code No: R05410105

IV B.Tech I Semester Supplimentary Examinations, March 2009 ANALYSIS AND DESIGN OF STRUCTURES FOR WIND AND EARTHQUAKE EFFECTS (Civil Engineering) Time: 3 hours Max Marks: 80 Answer any FIVE Questions All Questions carry equal marks ⋆⋆⋆⋆⋆ 1. (a) Define the terms i. Damping ii. Mode shape iii. Shear wall. (b) Write a detailed note on the hurricane KATRINA. Write the Date of formation, Highest wind speed, Fatalities, Extent of damage and Areas afftected. [8+8] 2. A platform of weight W = 340 kN is being supported by 4 equal columns which are clamped to the foundation as well as to the platform. Experimentally, it was determined that a static force of F = 12 kN kg applied horizontally to the platform produces a displacement of 0.10 cm. It is estimated that, damping in the structures is of the order of 4% of the critical damping. Determine (a) Undamped natural frequency (b) Absolute damping coefficient (c) Logarithmic decrement (d) The number of cycles and the time required for the amplitude of motion to be reduced to 0.01 cm. [16] 3. (a) Define Degrees of Freedom. Explain, how do you select them. (b) Formulate the Stiffness matrix for the frame shown in the figure 3.

[6+10]

Figure 3 4. (a) Explain, how the Architectural features affect buildings during earthquakes? (b) Write short notes on the following: 1 of 2

Set No. 2

Code No: R05410105 i. Design basis earthquake (DBE) ii. Maximum considered earthquake (MCE).

[8+8]

5. (a) Why are open ground storey buildings vulnerable in earthquakes? (b) Why is it desirable to design for the formation of plastic hinges in beams rather than columns in earthquake resistant design ? [8+8] 6. (a) Explain the various factors influncing the ductility. (b) A seven storey building is to be constructed at Hyderabad. The foundation to be provided is raft over the medium soil. Using I.S. specifications, determine the earthquake forces in the bottom three floors, if the structural framing system consists of moment resistant frames with steel bracing members designed for ductility. Live load on each floor = 4000 N/m2 Live load on roof = 1500 N/m2 Dead load of floor finish, slab and beam = 3000 N/m2 Height of each storey = 3.20 m Spacing of the columns in both directions = 4 m. [6+10] 7. (a) What are the limits placed on tensile reinforcement ratios in beams in earthquake resistant design? why are such limits enforced? (b) What are the salient features of the specifications of I.S. 13920 for the design and detailing of beams in earthquake resistant design? [8+8] 8. (a) Write a detailed notes on rigid frame shear walls. (b) What are the various methods available for the analysis & design of shear walls without openings ? Explain any one of them. [8+8] ⋆⋆⋆⋆⋆

2 of 2

Set No. 3

Code No: R05410105

IV B.Tech I Semester Supplimentary Examinations, March 2009 ANALYSIS AND DESIGN OF STRUCTURES FOR WIND AND EARTHQUAKE EFFECTS (Civil Engineering) Time: 3 hours Max Marks: 80 Answer any FIVE Questions All Questions carry equal marks ⋆⋆⋆⋆⋆ 1. (a) Write a detailed note on the instruments that measures earthquakes. (b) Write a detailed note on the hurricane KATRINA. Write the Date of formation, Highest wind speed, Fatalities, Extent of damage and Areas afftected. [8+8] 2. (a) List the various methods of evaluating the damping in SDOF systems. Explain any one method in detail. (b) What is Duhamel integral? Explain, how you evaluate the Duhamel integral, numerically, for an undamped system. [8+8] 3. (a) What are the lumped mass and consistent mass matrices? (b) Formulate the Stiffness matrix for the frame shown in the figure 3.

[6+10]

Figure 3 4. (a) A R.C. framed building is 45 m × 15 m in plan and 40 m in height consisting of stories 4 m in height. It is braced in the longitudinal direction by rigid frame action and by a R.C. infill wall in the transverse direction. Determine the design wind force on the framed building. Assume that the building is situated in terrain category 3 with basic wind speed of 50 m/sec in a fully developed velocity profile. (b) Explain the Static and Dynamic effects of wind on various types of structures. [8+8] 5. (a) What are the precautions to be taken at the design stage for ensuring a high quality structure? (b) Why are open ground storey buildings vulnerable to earthquakes?

1 of 2

[8+8]

Set No. 3

Code No: R05410105

6. (a) Draw the schematic c/s through a traditional stone masonry building. What are the various ways in which these buildings may suffer damage due to earthquakes. (b) A tall building is to be erected at the outskirts of Hyderabad. The terrain belongs to category 1 and the building is a class B structure. Determine the maximum lateral pressure (considering windward & leeward faces ) at a height of 40 m if the building has medium permeability. The height to width ratio of the building is 6.5 and the length to width ratio is 1.5. [8+8] 7. In a multi storey RCC frame building, a typical floor beam with 140 mm thick slab carries service negative bending moment and shear force of 500 kN.m and 360 kN respectively at the face of beam-column joint due to gravity and seismic loads. The size of the beam web has been fixed at 345 mm × 500 mm from architectural considerations. The effective cover to tension steel is 55mm. Design the beam section for adequate ductility. The materials used are M 20 concrete and Fe 415 steel. [16] 8. Design a shear wall of length 4.16 m and thickness 250 mm subjected to the following forces. Use M 25 grade concrete and Fe 415 grade steel. The wall is a high wall with the following loadings. Shown in figure 8.

Figure 8 S.No. Loading Axial force (kN ) Moment (kN. m) Shear (kN) 1. DL + LL 1900 600 20 2. Seismic load 250 4700 700 ⋆⋆⋆⋆⋆

2 of 2

[16]

Set No. 4

Code No: R05410105

IV B.Tech I Semester Supplimentary Examinations, March 2009 ANALYSIS AND DESIGN OF STRUCTURES FOR WIND AND EARTHQUAKE EFFECTS (Civil Engineering) Time: 3 hours Max Marks: 80 Answer any FIVE Questions All Questions carry equal marks ⋆⋆⋆⋆⋆ 1. (a) What are the characteristics of P-Waves & S-Waves? Sketch these waves. (b) Explain the “Elastic rebound theory” of earthquake generation.

[8+8]

2. A one-storey building is idealized as a rigid girder supported by weightless columns as shown in the figure 2. In order to evaluate the dynamic properties of this structure, a free vibration test is made, in which the roof system (rigid girder) is displaced laterally by a hydraulic jack and then released. During the jacking operation, it is observed that a force of 235 kN is required to displace the girder by 2 mm. After the instantaneous release of this initial displacement, the maximum displacement on the return swing is only 0.16 cm and the period of this displacement cycle is 1.40 sec.

Determine the following:

Figure 2

(a) Effective weight of the girder (b) Frequency of vibration (c) Logarithmic decrement, Damping ratio (d) Damping coefficient, Damped frequency (e) Amplitude after 6 cycles.

[16]

3. (a) Prove that the mode shapes are orthogonal with respect to mass matrix. (b) Determine the natural frequencies of vibration and corresponding Mode shapes for the 2-storied building frame shown in the figure 3. [4+12]

1 of 2

Set No. 4

Code No: R05410105

Figure 3 4. (a) An industrial building of 15 m height is being built at Hyderabad, near a hillock. The height of the hill is 160 m and the slope is 1 in 4. The building is proposed on the slope at a horizontal distance of 100 m from the base of the hill. Find the design wind pressure. (b) Explain the Static and Dynamic effects of wind on various types of structures. [8+8] 5. (a) Why are open ground storey buildings vulnerable in earthquakes? (b) Why is it desirable to design for the formation of plastic hinges in beams rather than columns in earthquake resistant design ? [8+8] 6. (a) Draw the schematic c/s through a traditional stone masonry building. What are the various ways in which these buildings may suffer damage due to earthquakes. (b) A tall building is to be erected at the outskirts of Hyderabad. The terrain belongs to category 1 and the building is a class B structure. Determine the maximum lateral pressure (considering windward & leeward faces ) at a height of 40 m if the building has medium permeability. The height to width ratio of the building is 6.5 and the length to width ratio is 1.5. [8+8] 7. (a) What is meant by ductility? Give a qualitative description and also describe briefly quantitative measures of ductility in reinforced concrete. (b) What are the salient features of the specifications of I.S. 13920 for the design and detailing of beams in earthquake resistant design? [8+8] 8. (a) Explain the differences between an ordinary wall and a shear wall in a reinforced concrete tall building, with regard to function, loading and design. (b) Explain the principle of shear wall analysis. How do you compute the stiffness of the wall? [8+8] ⋆⋆⋆⋆⋆

2 of 2

Set No. 1

Code No: R05410106

IV B.Tech I Semester Supplimentary Examinations, March 2009 INDUSTRIAL WASTE AND WASTE WATER MANAGEMENT (Civil Engineering) Time: 3 hours Max Marks: 80 Answer any FIVE Questions All Questions carry equal marks ⋆⋆⋆⋆⋆ 1. Draw the flow chart of treating waste water of a typical Food processing industry. [16] 2. What are the main differences between volume reduction and strength reduction? [16] 3. Draw a sketch and explain the process of disposal of waste water into an Ocean. [16] 4. What are the most common industrial waste water reuse applications? What are the most important issues to be addressed? [16] 5. (a) Explain the processing of raw cotton to finished cloth with the help of a flow diagram. (b) Describe the treatment of Viscose - Rayon waste.

][8+8]

6. Explain the effects of the following industrial effluent on aquatic environment when discharged without treatment (a) Nitrogenous fertilizer plant effluent (b) Molasses based distillary effluent (c) Dairy effluent.

[16]

7. Explain the Complete treatment of sugar mill waste with the help of a flow diagram. [16] 8. (a) Discuss the concept of common effluent treatment plant. (b) Discuss the operation and maintenpance problems of common effluent treatment plant. [8+8] ⋆⋆⋆⋆⋆

1 of 1

Set No. 2

Code No: R05410106

IV B.Tech I Semester Supplimentary Examinations, March 2009 INDUSTRIAL WASTE AND WASTE WATER MANAGEMENT (Civil Engineering) Time: 3 hours Max Marks: 80 Answer any FIVE Questions All Questions carry equal marks ⋆⋆⋆⋆⋆ 1. Discuss the treatment of waste water from Brewery industrial waste.

[16]

2. Write short notes on Volume reduction and Neutralization.

[16]

3. Explain the Oxygen sag curve in streams when industrial waste water is disposed into streams. [16] 4. Explain the various types of advanced treatments required for recirculation of Industrial waste water. [16] 5. (a) Describe the different stages involved in the tanning process with the help of a flow. diagram. (b) Explain the average compositions of Spent liquor and Combined wastes of a typical Indian Tannery. [8+8] 6. Explain the effects of the following industrial effluent on aquatic environment when discharged without treatment (a) Nitrogenous fertilizer plant effluent (b) Molasses based distillary effluent (c) Dairy effluent.

[16]

7. (a) Explain Rolling mills and their wastes in the manufacture of steel. Also explain the characteristics of wastes. (b) Discuss the characteristics of a typical power plant waste. Also discuss the effects of steel plants waste on receiving water. [8+8] 8. Explain how do you plan and data required for design of the common effluent treatment plant for the following industries: (a) A group of cotton textile dyeing units. (b) A group of chrome tanning industries. ⋆⋆⋆⋆⋆

1 of 1

[8+8]

Set No. 3

Code No: R05410106

IV B.Tech I Semester Supplimentary Examinations, March 2009 INDUSTRIAL WASTE AND WASTE WATER MANAGEMENT (Civil Engineering) Time: 3 hours Max Marks: 80 Answer any FIVE Questions All Questions carry equal marks ⋆⋆⋆⋆⋆ 1. Draw the flow chart of treating waste water of a typical Food processing industry. [16] 2. Write about the methods of joint treatment of waste water.

[16]

3. Explain the suitability of discharge of industrial waste into natural streams. [16] 4. What are the factors to be considered for the use of treated municipal waste water in industries? [16] 5. (a) Describe theories and practices of Textile mill waste treatment. (b) Explain the process of recovery of Zinc from Viscose Rayon wastes in detail. [8+8] 6. Explain the treatment of Fertilizer waste water in detail with the help of a flow diagram. [16] 7. (a) What is coke Oven plant in the manufacture of Steel. Also Discuss the source and types of waste water from coke Oven Plant. (b) Describe the treatment of coal washery waste by coagulation.

[8+8]

8. Explain how do you plan and data required for design of the common effluent treatment plant for the following industries: (a) A group of cotton textile dyeing units. (b) A group of chrome tanning industries. ⋆⋆⋆⋆⋆

1 of 1

[8+8]

Set No. 4

Code No: R05410106

IV B.Tech I Semester Supplimentary Examinations, March 2009 INDUSTRIAL WASTE AND WASTE WATER MANAGEMENT (Civil Engineering) Time: 3 hours Max Marks: 80 Answer any FIVE Questions All Questions carry equal marks ⋆⋆⋆⋆⋆ 1. What are the advantages and disadvantages of Boilers and cooling water?

[16]

2. What are the main differences between volume reduction and strength reduction? [16] 3. What are the advantages and disadvantages of disposal of industrial waste into streams? [16] 4. Explain the quality of water to be used for industrial processes.

[16]

5. (a) Discuss in detail, composition of wastes from cotton Textile, Woolen Textile and Viscose Rayon Plants. (b) Explain the manufacturing of Synthetic fabrics.

[8+8]

6. (a) What are the various sources of waste water from a molasses based disillary. Mention the typical characteristics waste water from each source. (b) Explain the various treatment process schemes of distillary effluent by means of neat process flow diagrams. [8+8] 7. (a) Write a detailed note on Blast Furnace waste water and its characteristics. (b) What are the effects of Steel Plants waste on receiving water. Also discuss disposal of waste pickle Liquor. [8+8] 8. What are the different media’s available for ultimate disposal of industrial waste waters ? Mention the typical tolerable limits of various physical and chemical parameters of effluent to be discharged in these media’s. [16] ⋆⋆⋆⋆⋆

1 of 1

Set No.1 Code No: R05 410107

IV B. Tech I Semester Supplementary Examinations, March 2009 Advanced Foundation Engineering (Civil Engineering) Time: 3 hours

Max. Marks: 80 Answer any FIVE questions All Questions carry equal marks

******** 1

A square footing of 2.5 x 2.5 m in dimension resting on sand at 1.25 m depth. The horizontal force is 0.15 times to the vertical force. Ground slope is 100 and base tilt is 100. Unit weight of the sand 16.5 kN/m3, c = 0 and φ = 300. The water table at great depth. Using Hansen theory, Compute the Ultimate Bearing Capacity of the footing. For φ = 300  Nc = 30.13, Nq = 18.4 and Nγ = 15.1 N B B B Shape factors Sc = 1 + q S q = 1 + tan φ and S r = 1 − 0.4 Nc L L L Depth factors dc = 1.0 + 0.4 k, dq = 1 + 2 tanφ(1-sinφ)k and dγ = 1.0; where k = D/B for D/B ≤ 1 and k = tan-1(D/B) for D/B > 1 (rad)   0 .5 H  Inclination factors ic = iq − , iq = 1 −   Nq − 1 V + A c cot φ f a   1 − iq

5

5

  0 .7 H  forη = 0 and iγ = 1 −  V + A c cot φ  f a   5

2. a

b 3. (a)

(b) 4. (a) (b)

 (0.7 − η / 450 )H   forη  0 where the η is the ground slope. iγ = 1 −   φ + V A c cot f a   Base factors: bc = 1-η/147, bq = exp(-2ηtanφ) and bγ = exp(-2.7ηtanφ) Ground factors: gc = 1-β/147 and gq =gγ = (1-tanβ)5. What are the different types of settlements occurred in soil with the foundation loads? and which settlements are predominant in what type of soils ? How do you estimate the settlement of a footing on sand using the results of a plate load test? How is the depth correction is applied? Explain the basic difference in the bearing capacity computation of shallow and deep foundation. How is skin friction and point resistance of a pile computed? A 30 cm square pile, 15 m long, is driven in a deposit of medium dense sand (φ = 360, Nφ = 42 and Nγ = 40). The unit weight of sand is 15 kN/m3. What is the allowable load with a factor of safety of 3? Assume lateral earth pressure coefficient = 0.6. Explain the outline of settlement of pile groups in sands A group of piles of 30 cm diameter is subjected to a net load of 2000 kN as shown in Fig. 1. compute the consolidation settlement of these pile group.

Q

γt = 16 kN/m2

3m

γt = 19 kN/m2 12 m

13 m γsat = 21 kN/m2 φ = 300 e0 = 0.75 2.5 m

10 m

5. (a)

Clay γsat = 20 kN/m2 cc = 0.20 e0 = 0.65

Fig. 1

Explain the difference between the active, at-rest and passive earth conditions

(b)

What are the assumptions made in Coulomb earth pressure theory?

(c)

Which of the three earth pressure conditions should be used to design a rigid basement wall. A Cylindrical Well has an external diameter of 8 m and is sunk into sand to a depth of 21 m below the scour level. The well is acted upon by a moment of 40 MN-m about the scour level due to a horizontal force of 20 MN. Check whether the safe assuming (a) it rotates about a point above the base, and (b) it rotates about the base. γsat = 19.8 kN/m3, φ = 300. Use Terzaghi’s Approach, with a factor of safety of 2 against passive resistance. Describe the equivalent beam method beam method for the analysis of an anchored sheet pile.

6.

7. (a)

(b)

Classify sheet piles according to their material. Discuss their relative advantages and

disadvantages.

8. (a)

(b)

Indicate and describe the any two methods of finding the swell pressure in expansive soils. Define terms, ‘free swell’, differential swell, swelling pressure, field moisture content and equilibrium moisture content

Set No.2 Code No: R05 410107

IV B. Tech I Semester Supplementary Examinations, March 2009 Advanced Foundation Engineering (Civil Engineering) Time: 3 hours

Max. Marks: 80 Answer any FIVE questions All Questions carry equal marks

******** 1

A footing of B = 3 X L = 6 m is to be placed on a two layered clay deposit as shown in Fig. 1 Estimate the ultimate bearing capacity.

Su = 77 kPa γ = 17.26 kN/m3 1.83 m

1.22 m

Fig. 1 2

Su = 115kPa

A square foundation of 2.5 m resting on a sand deposit at 2.0 m depth. The total pressure at the foundation level is 200 kN/m2. the variation of cone penetration resistance with depth is simplified as shown in Fig. 2. Determine the settlement of the foundation 6 years after the construction. Use Schemrtmann approach. The ground water table is at great depth. Unit weight of the soil 17 kN/m2.

Depth

qc kN/m3 X 103

2m

Foundation Level

3

3m 4 6m

8

7m 8m

2 Fig. 2

3 (a) (b)

Give the design specifications of the Under-reamed piles. A symmetrical 16 pile group in soft clay, with unconfined compressive strength of 50 kN/m2 is to be used as foundation for a column. The piles are 400 mm in diameters and 10 m long and spaced at 1.2 m centers. Determine the maximum load the group can carry with the piles failing (i) individually and (ii) as a block.

4. (a)

Discuss the method of Reese and Metlock for the determination of the deflection of a laterally loaded pile.

(b)

Load versus Lateral displacement data is given in the following table. Load Settleme nt

0 0

16.67 1

21.53 3

22.72 5

23.33 7

Load Settleme nt

23.1 10

24.19 15

24.51 25

24.65 35

24.75 50

23.53 8

Using the P-Y Curve, compute the lateral capacity of a pile of diameter 30 mm and installed in deep clay. 5 (a) (b) 6 (a) (b)

Explain how drainage is provided in retaining walls. What are the different modes of failure of the retaining walls? Explain with neat sketches. What are the requirements for the horizontal cross section of the well foundation? Sketch some of the commonly used shapes. What is Pneumatic caisson? Sketch a Pneumatic caisson and briefly outline how it is constructed.

7 (a) (b)

8

Sketch a typical section of a braced cut and show the various components A cut 4.5 m wide, 9.0m deep is proposed in a cohesive deposit (γc = 18 kN/m3, Cu = 50 kN/m3 ). The first row and lost row of strut are to be located at the below ground surface and 1.5 m above the bottom of the cut respectively. If the spacing between the strut is 2 m. Determine the loads on the struts. Assume the center to center spacing of the strut along the length of the cut is 2.0 m Describe briefly any Three of the following (a) (b) (c) (d)

Sand Cushions CNS layer Under reamed piles Moisture Variation in expansive soil with depth and its importance in design of foundations on such soils.

Set No.3 Code No: R05 410107

IV B. Tech I Semester Supplementary Examinations, March 2009 Advanced Foundation Engineering (Civil Engineering) Time: 3 hours

Max. Marks: 80 Answer any FIVE questions All Questions carry equal marks

******** 1

A square footing of 1.5 x 1.5 m in dimension resting on sand at 1.5 m depth. The horizontal force is 0.1 times to the vertical force. Ground slope is 100 and base tilt is 100. Unit weight of the sand 17.4 kN/m3, c = 0 and φ = 300. The water table at great depth. Using Vesic, Compute the Ultimate Bearing Capacity of the footing. For φ = 300  Nc = 30.13, Nq = 18.4 and Nγ = 22.4 N B B B S q = 1 + tan φ and S r = 1 − 0.4 Shape factors Sc = 1 + q Nc L L L Depth factors dc = 1.0 + 0.4 k, dq = 1 + 2 tanφ(1-sinφ)k and dγ = 1.0; where k = D/B for D/B ≤ 1 and k = tan-1(D/B) for D/B > 1 (rad) 

H



m

2+ B/L

 where m = Inclination factors, iq = 1 − H is parallel to B  1+ B / L  V + A f ca cot φ    H  iγ = 1 −  V + A f ca cot φ   

2. (a)

(b) 3 (a)

m +1

Base factors: bc = 1-η/147, bq = bγ = (1-ηtanφ)2 Ground factors: gc = 1-β/147 and gq =gγ = (1-tanβ)2. A foundation 6 m X 3 m, carrying a uniform pressure of 200 kN/m2, is located at a depth of 1.5 m in a layer of clay 4 m thick for which the value of Eu is 35 MN/m2. The layer is underlain by a second clay layer 12 m thick for which the value of Eu is 80 MN/m2. A hard stratum lies below the second layer. Determine the average immediate settlement under the foundation For H/B = 2 and L/B  µ1 = 0.60 H/B = 6 and L/B  µ2 = 0.85 For D/B = 0.5 and L/B = 2  µ0 = 0.94. Differentiate between total settlement and differential settlement. What are the harmful effects of differential settlements on structures? What are the possible remedial measures? 3. (a) Differentiate critically in principles in deriving the Engineering News formula and Hiley’s formula for Pile load capacity.

(b)

What are the basis on which the dynamic pile formulae are derived?.

(c )

Discuss critically the limitations of dynamic pile formulas.

(d)

4 (a)

What will be the penetration per blow of a pile, which must be obtained in driving with a 3t steam hammer falling through 1 m allowable load, is 25 tonnes?

Explain the outline of settlement of pile groups in sands (5) A group of piles of 30 cm diameter is subjected to a net load of 2000 kN as shown in Fig. 1. Estimate the negative frictional force on the group of piles, if negative skin frictional force is developed. Q

γt = 15 kN/m2

3m

γt = 16.5 kN/m2 15 m

16 m

Fresh fill

γsat = 21 kN/m2 φ = 300 e0 = 0.75 2.5 m

10 m Fig. 1

5

6 (a)

A gravity retaining wall is 4.5 m high and it carries a level back fill with a uniform surcharge of 125 kN/m2. The water table is 1.5 m below the top of the fill and the moisture content of the fill above the water table is 15%. Back fill properties: Dry density – 18.0 kN/m3. Angle of internal friction = 300. Specific gravity of solids = 2.60. porosity of the backfill – 40%. Calculate the earth pressure on the well neglecting wall friction. The surface of the wall by neglecting wall friction. The surface of the wall in contact with the fill is vertical. Compute the stability of the retaining wall. Sketch a completed well foundation for a Bridge pier. Indicate the various components

and their functions. (b) 7

How do you analyze the well for its lateral stability? What is the design pressure diagram for a braced cut due to earth pressure diagram for a braced cut due to earth pressure when the soil is (i) dense sand, (ii) loose sand and clay ? Find the thrust loads for the case shown in Fig. 2. The horizontal spacing of struts is 5 m. If the sheeting are driven below the bottom of the cut how will that affect the stability of the cut? GL 2.0 m γ = 18 kN/m3 C = 1.5 T/m2

2.0 m 2.0 m 2.0 m Fig. 2 8 (a)

(b)

How the sand Cushion is effective in arresting of heave. Explain in detail. Is it effective for many years ? If Yes Explain in detail.

What is the mechanism of granular pile anchor, while arresting heave ? Explain with sketches in detail

Set No.4 Code No: R05 410107

IV B. Tech I Semester Supplementary Examinations, March 2009 Advanced Foundation Engineering (Civil Engineering) Time: 3 hours

Max. Marks: 80 Answer any FIVE questions All Questions carry equal marks

******** 1

1.

A square footing of 1.5 x 1.5 m in dimension resting on sand at 1.5 m depth. The horizontal force is 0.1 times to the vertical force. Ground slope is 100 and base tilt is 100. Unit weight of the sand 17.4 kN/m3, c = 0 and φ = 300. The water table at great depth. Using Hansen theory, Compute the Ultimate Bearing Capacity of the footing. For φ = 300  Nc = 30.13, Nq = 18.4 and Nγ = 15.1 N B B B Shape factors Sc = 1 + q S q = 1 + tan φ and S r = 1 − 0.4 Nc L L L Depth factors dc = 1.0 + 0.4 k, dq = 1 + 2 tanφ(1-sinφ)k and dγ = 1.0; where k = D/B for D/B ≤ 1 and k = tan-1(D/B) for D/B > 1 (rad)   0 .5 H  Inclination factors ic = iq − , iq = 1 −  V + A c cot φ  Nq − 1 f a   1 − iq

5

5

  0 .7 H  forη = 0 and iγ = 1 −  V + A c cot φ  f a   5

2. a

 (0.7 − η / 450 )H   forη  0 where the η is the ground slope. iγ = 1 −   φ V + A c cot f a   Base factors: bc = 1-η/147, bq = exp(-2ηtanφ) and bγ = exp(-2.7ηtanφ) Ground factors: gc = 1-β/147 and gq =gγ = (1-tanβ)5. A foundation 4 m X 2 m, carrying a uniform pressure of 150 kN/m2, is located at a depth of 1 m in a layer of clay 5 m thick for which the value of Eu is 40 MN/m2. The layer is underlain by a second clay layer 8 m thick for which the value of Eu is 75 MN/m2. A hard stratum lies below the second layer. Determine the average immediate settlement under the foundation For H/B = 2 and L/B  µ1 = 0.60 H/B = 6 and L/B  µ2 = 0.85 For D/B = 0.5 and L/B = 2  µ0 = 0.94.

b

Differentiate between total settlement and differential settlement. What are the harmful effects of differential settlements on structures? What are the possible remedial measures?

3. (a) (b)

4. (a)

(b)

5 (a)

What is negative skin friction? How is it computed for a group of piles in clay? From the cyclic pile load test the following data is obtained.

Load in Elastic (T) Settlement 0 0 1 16.96667 3 22.32857 5 24.22727 7 25.43333 8 25.92941 10 26.80952 15 28.69355 25 32.0098 35 35.14789 50 39.75248 Compute Skin frictional resistance and Base resistance Discuss the method of Reese and Metlock for the determination of the deflection of a laterally loaded pile. A 20 m long RCC Pile is installed in a deposit of uniform sand. The pile head is subjected to a horizontal force of 30 kN. Assuming the coefficient of subgrade modulus of 12.6 X 106 N/m3. Calculate the deflection of pile head. What will be the change in deflection, if the pile head is fixed? Assume EI of pile as 4.2 X 107 Nm2. At the top of the pile the Reese and Metlock coefficients for deflection are Ay = 2.435 and By = 1.023. . Find the location and magnitude of the thrust on the wall for the Fig. 1 shown below

4.0 T 2.0 m

3.0 m 150

200

5.0 m

Fig. 1

6 (a)

Under what circumstances is a pneumatic caisson preferred? What are the safety precautions to be fallowed in working with a pneumatic caisson?.

(b)

What is grip length of well? What are the considerations in determination of grip length?

(c) What are ‘Tilts and Shifts’? What are the remedial measures to control these? 7 (a) (b)

What are the forces acting on a bulk head? Compute the embedment length D of the sheet pile wall shown in Fig. 2

6m

3m 3m

γ = 18 kN/m3 φ = 300

γsat = 20 kN/m3 φ = 300

D

Fig. 2

GWT

8

Describe briefly any Three of the following (a) (b) (c) (d)

Granular Pile anchors Swell pressure and Swell Potential Foundation Problems in expansive soils Foundation practices in Expansive soils

Set No.1 Code No: R05 410205

IV B. Tech I Semester Supplementary Examinations, March 2009 Advanced Control Systems (Electrical & Electronics Engineering) Time: 3 hours

Max. Marks: 80

Answer any FIVE questions All Questions carry equal marks ******** 1. a) Derive the equations for the solution of state equation by replace transform method.

b) Given that A1 =

, A2 =

,A=

,Compute eAt.

2. a) State and explain the controllability for continuous time systems. b) The state model of a system is given by x1 0 0 1 x1 0 u, x2 = -2 -3 0 x2 + 2 y= x3

0

2

-3

x3

0

x1 x2 x3

convert the state model to controllable phase variable form. 3. Explain the describing function of saturation non – linearity. 4. a) Discuss the singular points. b) Explain the procedure to draw phase trajectory by delta method. 5. a) Determain whether the following quadratic form is positive definite. Q = x21 + 4x22 +x23 + 2 x1x2 – 6x2x3 – 2x1x3 b) Write a lyapunov function for the system. = Determine the stability of the origin of the system . 6. a) Explain the effect of feedback on controllability and observability. b) Design a full order state observes for the given state model. Given

A=

,

C=

And given values are µ 1 = - 5 and µ 2 = - 5 7. Illustrate with an example the problem with terminal time t1 free and x (t1) fixed. 8. Write short notes on the following for optimal control .

a) b) c)

Minimum energy problem. Minimum fuel problem. Tracking problem.

-------------------

Set No.2 Code No: R05 410205

IV B. Tech I Semester Supplementary Examinations, March 2009 Advanced Control Systems (Electrical & Electronics Engineering) Time: 3 hours

Max. Marks: 80

Answer any FIVE questions All Questions carry equal marks ******** 1. a) Explain the Properties of State transition matrix. b) Determine the canonical state model of the system, whose transfer function is T(s) = 2. a) Explain the principle of duality. b) Is the following system completely state controllable. 0 1 0 x1 0 (t) =

0

0

1

x2

+ 0

-6

-11 -6

x3

1

u(t), y(t) = 2 0 9 1

x1 x2 x3

3. Explain the describing function of dead-zone and saturation non linearity. 4. Construct a phase trajectory by delta method for nonlinear system represented by the + 4x = 0. Choose the initial conditions as x (0) = 1.0 and (0) =0 differential equation, + 4 5. a) State and explain the lyapunov instability theorem . b) By use of krasovskii’s theorem, examine the stability of the equilibrium state x=0 of the following system . 1

2

= - x1 = x1 - x 2 - x2 3

6. a) Explain the design of state feedback control through pole placement. b) Consider the system described by the state model = Ax and y = cx where A =

, c =

Obtain the state observer gain matrix using all these methods. The desired given values for the observer matrix are µ 1 = - 5, µ 2 = - 5

7. a) Briefly discuss the minimization of functional. b) Given

= - x + 4 , x(0) = x0 , x(2) = x1 find u* that minimizes J =

8. Write short notes on the following for optimal control a) Minimum time problem b) Minimum energy problem c) State regulator problem.

Set No.3 Code No: R05 410205

IV B. Tech I Semester Supplementary Examinations, March 2009 Advanced Control Systems (Electrical & Electronics Engineering) Time: 3 hours

Max. Marks: 80

Answer any FIVE questions All Questions carry equal marks ******** 1. a) Explain the state space representation using phase variables b) Construct a state model for the system characterized by the differential equation

2. write the state equations for the system shown in figure in which x1, x2, x3 constitute the state vector. Determine whether the system is completely controllable and observable T=1sec

u(s)

X1(3)

X2(s) X3(s)

s

3. a) Explain the types of non linear systems b) Discuss the describing function 4. a linear second order servo is described by the equation +2 where = 0.15,

+wn2e=0

=1 rad/sec, e(0)=1.5, (0)=0.

Determine the singular point. Construct the phase trajectory, using the method of Isoclines 5. For the system, = [x], find a suitable lyapunov function. Find an upper bound on time 1 that x(0)= 1 to with in the area defined by x12+x22=0.1 6. Consider a linear system described by the transfer function

. Design a feedback

controller. With a state feedback so that the closed loop poles are placed at – 2, -1 j1 7. Illustrate with an example the problem with terminal time t1 free, x(t1) specified 8. Briefly discuss the formation of the optional control problem. 9.

Set No.4 Code No: R05 410205

IV B. Tech I Semester Supplementary Examinations, March 2009 Advanced Control Systems (Electrical & Electronics Engineering) Time: 3 hours

Max. Marks: 80

Answer any FIVE questions All Questions carry equal marks ******** 1

A feedback system has a closed loop transfer function

Construct three different state models for this system and give block diagram representation for each state model. 2

a) State and explain the observability for continuous time system. b) Consider the following state equations. =

x

+

u , y=

x

Check the controllability and observability. 3 4

a) Explain the characteristics of non-linear systems. b) Briefly discuss the type of nonlinearities. Consider a system with an ideal relay as shown in figure.Determine the singular point. Construct phase trajectories, corresponding to intial contion C(o)=2, (o) =1 Take r=2volts , n=1.2volts

e

r

u M

ee

-

u

-M

5

a) State and explain the lyapunov stability theorem b) check the stability of the system described by 2 1= -x1+2x ,x2 2= -x2, by using variable gradient method

6

a) Explain the reduced order observer with neat block diagram

C

b) A regulator system has a plant , by use of state feedback control u=-kx, it is desired to place the closed loop poles at -2 ±j2 7 8

and -10. Determine the necessary state feedback gain matrix.

Illustrate with an example the problem with terminal time t1 and x(t1) free a) What are the steps required for the solution of an optimal control problem b) Explain the state regulator problem in optimal control theory.

Set No. 1

Code No: R05410305

IV B.Tech I Semester Supplimentary Examinations, March 2009 REFRIGERATION AND AIR CONDITIONING (Mechanical Engineering) Time: 3 hours Max Marks: 80 Answer any FIVE Questions All Questions carry equal marks ⋆⋆⋆⋆⋆ 1. In an aeroplane a simple air refrigeration is used. The main compressor delivers the air at 5 bar and 2000 C. The bled air taken from compressor is passed through a heat exchanger, cooled with the help of ram air so that the temperature of air leaving the heat exchanger is 450 C and the pressure is 4.5 bar. The cooling turbine drives the exhaust fan which is used to force the ram air through the heat exchanger. The air leaving the heat exchanger passes through the cooling turbine and then supplied to cabin at 1 bar. The pressure loss between the cooling turbine and cabin is 0.2 bar. If the rate of flow of air through the cooling turbine is 20kg/min, determine the following: (a) The temperature of the air leaving the expander, (b) the power delivered to the ram air which is passed through the heat exchanger and (c) the refrigeration load in tones when the temperature of the air leaving the cabin is limited to 250 C. Assume that the isentropic efficiency of the cooling turbine is 75% and no loss of heat from air between the cooling turbine and cabin. Take γ = 1.4 and cp =1kJ/kg.K. [16] 2. (a) What is the object of under cooling? (b) In an ammonia vapour compression refrigerator, the temperature of refrigerator is -100 C and temperature of NH3 coming out of the compressor is 30o C. The vapour is condensed in the condenser at 300 C. Find the theoritical C O P of the cycle when the vapour at the end of compression is 0.9 dry. Latent heat of NH3 at 300 C = 1442 Kj / kg Specific heat of liquid NH3 = 4.7 Kj /kg [6+10] 3. (a) Describe the effect of section temperature on the refrigerating capacity and brake power of a reciprocating commpressor. (b) What problems does lubricating oil cause in the evaporator?

[8+8]

4. (a) Make a comparative list between a vapour - absorption system and a compression system. (b) Explain the Electro-lux refrigeration system with a neat sketch. What is the purpose of hydrogen in it? [16]

1 of 2

Set No. 1

Code No: R05410305

5. A steam ejector water vapour refrigeration system is supplied with motive steam at 9 bar and 97% dry. Flash chamber water temperature is 60 C. Make-up water is supplied at 23 0 C and condenser pressure is 55 mm of Hg absolute. Take Nozzle efficiency = 92%, Entrainment efficiency = 73%, Thermo-compressor efficiency = 80%. The quality of the mixture of motive steam and flashed vapour at the beginning of compression is 91% dry and saturated. Determine the following: (a) kg of motive steam required per kg of flashed vapour. (b) Quality of vapour flashed from flash-chamber. (c) Refrigeration effect per kg of flashed vapour. (d) If the load on the system is 100 tons of refrigeration, find the weight of steam required per hour. [16] 6. An air-conditioned system is to be designed for a hall of 150 seating when the following conditions are given: Atmospheric conditions are 270 C DBT and 55% R.H., Indoor conditions are 200 C DBT and 65% R.H., Amount of free air supplied is 0.5 m3 /min/person, The required condition is achieved first by chemical dehumidification and after that by sensible cooling. Find the following: (a) DBT of the air leaving the dehumidifier (b) The quantity of water vapour removed in the dehumidifier per hour (c) The capacity of the cooling coil in tons of refrigeration. If the bypass factor of the coil is 0.23, find the surface temperature of the coil. [16] 7. (a) Explain the difference between comfort air-conditioning and industrial airconditioning. (b) Define the term “Effective Temperature” and explain its importance in airconditioning system. Describe the factors which affect effective temperature. [6+10] 8. (a) What are the purposes of automatic controls in an air conditioning system? (b) Explain the function of basic components of automatic control circuit of an air conditioning system. [6+10] ⋆⋆⋆⋆⋆

2 of 2

Set No. 2

Code No: R05410305

IV B.Tech I Semester Supplimentary Examinations, March 2009 REFRIGERATION AND AIR CONDITIONING (Mechanical Engineering) Time: 3 hours Max Marks: 80 Answer any FIVE Questions All Questions carry equal marks ⋆⋆⋆⋆⋆ 1. (a) Describe Briefly the simple air craft refrigeration system. (b) Sketch a actual air craft refrigeration system showing various control units. [8+8] 2. (a) Explain the construction of T-S and P- h diagrams and state why P- h diagram is more suitable for calculations over T-S diagram. (b) A refrigerating machine using NH3 operates between the temperature limits of -150 C and 300 C. Find the C O P of the system. Also find the corresponding value for a reversed carnot cycle operating between the same temperatures. The properties of NH3 are given below: [8+8] Pressure(bar) Temperature0 C 2.41 11.895

-5 30

Liquid Vapour hf(Kj /Kg) Sf(Kj /Kg-K) hg(Kj /Kg) Sg(Kj /Kg-K) 351 3.95 1667.5 9.05 562 4.69 1711 8.48

3. (a) Name the different refrigerants generally used. (b) What is an azeotrope? Give some examples to indicate its importance. (c) Explain, with the help of a neat sketch, the principle of operation of a single stage, single acting reciprocating commpressior. [4+4+8] 4. (a) Make a comparative list between a vapour - absorption system and a compression system. (b) Explain the Electro-lux refrigeration system with a neat sketch. What is the purpose of hydrogen in it? [16] 5. A steam jet refrigeration unit is supplied with steam at 8 bar dry and saturated and it maintains the flash chamber water at 50 C, when the makeup water is available at 220 C, and the condenser is at 4.5 cm of Hg absolute. The quality of flashed vapour and steam after mixing is 0.9. Assuming nozzle efficiency 90%, the entrainment efficiency 63% and the diffuser efficiency 79%. Determine: (a) mass of motive steam/kg of flashed vapour (b) cooling effect produced in kJ/kg of flashed vapour (c) quality of vapour flashed from flash chamber (d) mass of motive steam per ton of cooling 1 of 2

Set No. 2

Code No: R05410305 (e) volume of vapour flashed per ton of cooling.

[16]

6. The following two indoor conditions give the same comfort when the out-door conditions are 380 C and 62% R H. (a) 200 C DBT and 60% R.H. (b) 240 C DBT and 50% R.H. The required conditions are achieved first by cooling and dehumidifying and then by heating. The dew-point temperature of the cooling coil is 80 C. If the supply of free air to the air-conditioning system is 450 m3 /min., then find the cost of the plant for 24-hours working for both comfort conditions. Also find the by-pass factors of the cooling coils in both cases if the Cost of cooling = 50 Paise per ton of refrigeration and the Cost of heating = 15 Paise per kWh. [16] 7. A class room is to be air-conditioned for the following given summer conditions. The data collected is given as follows: Size of class-room: 18 × 12 × 6 m, out-door conditions: 420 C DBT and 52% R.H., required comfort condition: 180 C DBT and 58% R.H., seating capacity: 45, sensible heat in the room excluding infiltrated load: 40,000 kJ/hr, sensible heat load from other sources: 8000 kJ/hr, lighting load: l2 tubes of 80 watts, infiltrated air: 27 m3 /min. If 35% air is taken from outside and remaining is recirculated then find the following: (a) Capacity of the cooling coil in tons of refrigeration and its bypass factor (b) Capacity of the blower in m3/min. Assume DPT of the coil 60 C.

[16]

8. (a) Which type of air cleaner would be selected for removing very small dirt particles and smoke from the air? Explain its working principle. (b) Explain the advantages and disadvantages of viscous filters over dry filters. [8+8] ⋆⋆⋆⋆⋆

2 of 2

Set No. 3

Code No: R05410305

IV B.Tech I Semester Supplimentary Examinations, March 2009 REFRIGERATION AND AIR CONDITIONING (Mechanical Engineering) Time: 3 hours Max Marks: 80 Answer any FIVE Questions All Questions carry equal marks ⋆⋆⋆⋆⋆ 1. In an aeroplane a simple air refrigeration is used. The main compressor delivers the air at 5 bar and 2000 C. The bled air taken from compressor is passed through a heat exchanger, cooled with the help of ram air so that the temperature of air leaving the heat exchanger is 450 C and the pressure is 4.5 bar. The cooling turbine drives the exhaust fan which is used to force the ram air through the heat exchanger. The air leaving the heat exchanger passes through the cooling turbine and then supplied to cabin at 1 bar. The pressure loss between the cooling turbine and cabin is 0.2 bar. If the rate of flow of air through the cooling turbine is 20kg/min, determine the following: (a) The temperature of the air leaving the expander, (b) the power delivered to the ram air which is passed through the heat exchanger and (c) the refrigeration load in tones when the temperature of the air leaving the cabin is limited to 250 C. Assume that the isentropic efficiency of the cooling turbine is 75% and no loss of heat from air between the cooling turbine and cabin. Take γ = 1.4 and cp =1kJ/kg.K. [16] 2. (a) Explain the importance of super heating vapours before suction to compressor, How does it affect C.O.P. of the system. (b) A refrigerator machine uses R-12 as the working fluid. The temperature of R-12 in the evaporator coil is -50 C, and the gas leaves the compressor as dry saturated at a temperature of 400 C. The mean specific heat of liquid R-12 between the above temperatures is 0.963 kj/kg0 k. Enthalpy of evaporation at 400 C is 203.2 kj/kg. Neglecting losses, find the C.O.P. [8+8] 3. (a) With the help of neat sketch explain the working of a double tube condenser. Discuss its limitations. (b) Explain evaporative condenser and discuss its applications.

[6+10]

4. (a) Make a comparative list between a vapour - absorption system and a compression system. (b) Explain the Electro-lux refrigeration system with a neat sketch. What is the purpose of hydrogen in it? [16] 5. (a) List out the merits and demerits of thermo-electric refrigeration system over other refrigeration systems. 1 of 2

Set No. 3

Code No: R05410305

(b) What are the various applications of thermo-electric refrigerator.

[8+8]

6. (a) Show the following processes on the skeleton psychrometric chart : i. Dehumidification of moist air by cooling ii. Adiabatic mixing of two air streams (b) 1 kg of air at 220 C dry bulb temperature and 45% relative humidity is mixed adiabatically with 2.5 kg of air at 450 C dry bulb temperature and 45% relative humidity. Find specific humidity and enthalpy of the final condition of air. [6+10] 7. The following data refer to summer air conditioning of a restaurant : Inside design conditions = 250 C DBT and 190 C C WBT, outside design conditions = 360 C C DBT and 250 C C WBT, sensible heat load = 1,30,000 kJ/h, latent heat load= 50 000 kJ/h, the outside air is supplied at the rate of 23 m3 /min directly into the room through ventilators and by infiltration. The outside air to be conditioned is passed through a cooling coil which has an apparatus dew point of 100 C and 58% of the total air is recirculated from the conditioned space and mixed with conditioned air after the cooling coil. Find: (a) condition of air after the cooling coil before mixing with recirculated air (b) condition of air entering the restaurant (c) mass of fresh air entering the cooling coil (d) by-pass factor of the cooling coil (e) total refrigeration load of the cooling coil.

[16]

8. (a) What are the purposes of automatic controls in an air conditioning system? (b) Explain the function of basic components of automatic control circuit of an air conditioning system. [6+10] ⋆⋆⋆⋆⋆

2 of 2

Set No. 4

Code No: R05410305

IV B.Tech I Semester Supplimentary Examinations, March 2009 REFRIGERATION AND AIR CONDITIONING (Mechanical Engineering) Time: 3 hours Max Marks: 80 Answer any FIVE Questions All Questions carry equal marks ⋆⋆⋆⋆⋆ 1. (a) A Carnot refrigerator operates between the temperatures of -500 C and 500 C. Determine the C.O.P. of the refrigerator. If the C.O.P. is to be made 4 by changing the temperatures such that increase (or) decrease in upper temperature is equal to decrease (or) increase in lower temperature, determine the new temperatures. (b) Compare the various air cooling systems used for air crafts.

[16]

2. A food storage locker requires requires a refrigeration capacity of 12TR and works between the evaporating temperature of -80 C and condensing temperature of 300 C. The refrigerant R-12 is sub cooled by 50 C before entry to expansion valve and the vapour is superheated to -20 C before leaving the evaporator coils. Assuming a two cylinder, single acting compressor operating at 1000 rpm with stroke equal to 1.5 times the bore, determine: (a) C O P (b) Theoretical power per tonne of refrigeration and (c) bore and stroke of compressor when: i. there is no clearance and ii. there is a clearance of 2%. Use the following data for R-12. Saturation temperature. Pressure 0 (bar) C -8 30

2.354 7.451

Enthalpy Enthalpy Specific Volume kJ/kg kJ/kg.K m3 /kg Liquid Vapour Liquid Vapour 28.72 184.07 0.1149 0.7007 0.0790 64.59 199.62 0.2400 0.6853 0.0235

The specific heat of liquid R-12 is 1.235 kJ/kg.K and of vapour R-12 is 0.733kJ/kg.K. [16] 3. (a) With the help of neat sketch explain the working of a double tube condenser. Discuss its limitations. (b) Explain evaporative condenser and discuss its applications.

[6+10]

4. (a) What is the basic function of a compressor in vapour commpression refrigeration systems? How this function is achieved in vapour absorption refrigeration system?

1 of 2

Set No. 4

Code No: R05410305

(b) Draw a neat diagram of lithiemm bromide water absorption system and explain its working. List the major field of applications of this systems. [8+8] 5. In a steam jet refrigeration installation, the steam is available at 5.5 bar dry and saturated and chilled water flash chamber is kept at 6 cm of Hg. The mixture of steam and flashed vapour at entry to the diffuser has a quality of 0.89. Assuming nozzle efficiency 91%, mixing efficiency 66% and Diffuser efficiency 76% and make up water at 270 C, Determine: (a) mass of motive steam per kg of flashed vapour (b) mass of motive steam per ton of refrigerating effect (c) the coefficient of performance.

[16]

6. (a) Define degree of saturation and relative humidity. (b) A sling psychrometer reads 400 C DBT and 260 C WBT, calculate the following: i. ii. iii. iv. v.

Specific humidity Relative humidity Dew-point temperatur Enthalpy of mixture Specific volume of the mixture Assume pressure of atmospheric air to be 1.033 bar.

[6+10]

7. (a) Explain how does the body attempt to compensate for a warm environment approaching body temperature? (b) Draw and explain the comfort chart and zone of comfort for year-round airconditioning. [6+10] 8. Explain the working of heat pump with neat diagram when used for (a) Concentrating the juices (b) Purification of water.

[16] ⋆⋆⋆⋆⋆

2 of 2

Set No.1 Code No: R05 410306

IV B. Tech I Semester Supplementary Examinations, March 2009 Computational Fluid Dynamics (Mechanical Engineering) Time: 3 hours

Max. Marks: 80

Answer any FIVE questions All Questions carry equal marks ********

1.

Explain the direct method for matrix inverse? Briefly explain partial pivoting and LU decomposition used in direct method.

2.

A steel rod of length, 50 cm, diameter, 2 cm and thermal conductivity, 55 W/m-K is kept at a temperature of 1500C at its base. The fin is exposed to fluid of heat transfer coefficient, 15 W/m2K and temperature, 200C. The tip of the rod is kept at 500C. Obtain the temperature distribution for the fin using finite difference method and also calculate what amount of heat is lost from the surface maintained at 500C.

3.

Explain the term consistency in numerical schemes and in detail explain the behavior of errors and analyze them,

4.

Explain incompressible Reynolds Averaging Navier Stokes equation and write a note on averaging of property.

5.

Using Von Neumann Stability Analysis, obtain the criterion for stability of Explicit scheme for discretization of 1D parabolic equation.

6.

Derive the 2-D momentum equation to obtain the dimensionless form of it.

7.

Define Vorticity? How the pressure gradient term is eliminated from the momentum equations using Vorticity – Stream Function method? What are the disadvantages of this method in determination flow field.

8.

Write a short notes on: (a) (b) (c)

Upwind scheme in 1D convection – diffusion problem Implicit scheme in transient heat conduction Staggered grid in modeling of 1D convection – diffusion problem

Set No.2 Code No: R05 410306

IV B. Tech I Semester Supplementary Examinations, March 2009 Computational Fluid Dynamics (Mechanical Engineering) Time: 3 hours

Max. Marks: 80

Answer any FIVE questions All Questions carry equal marks ******** 1. Give the classification of linear PDEs. What are the different discretization schemes that are used to solve Parabolic equations. Compare them. 2. A steel rod of length, 50 cm, diameter, 2 cm and thermal conductivity, 55 W/m-K is kept at a temperature of 1500C at its base. The fin is exposed to fluid of heat transfer coefficient, 15 W/m2K and temperature, 200C. The tip of the rod is kept at 500C. Obtain the temperature distribution for the fin using finite difference method and also calculate what amount of heat is lost from the surface maintained at 500C. 3. Consider a square slab of side, L. The right, left and bottom surfaces of the slab are kept at 1000C, while the top surface is kept at 5000C. The thermal conductivity of the slab is 2 W/mK. Taking the mesh size as L/3, obtain the finite difference temperature distribution of slab. 4. What are different types of errors that occur during the discretization of PDEs? Using the Von Neumann Stability analysis, obtain the stability criterion for the explicit formulation of Parabolic equations. 5. Define Vorticity? How the pressure gradient term is eliminated from the momentum equations using Vorticity – Stream Function method? What are the disadvantages of this method in determination flow field. 6. What are the basic parameters used to assess interpolation schemes in finite volume formulation of Convection – Diffusion problems? 7. What are different schemes of interpolation used in the discretization of 1D Convection – Diffusion problem. Write about Central Difference Scheme in detail and compare them. 8. Write short notes on any two of the following: a. What are the four basic rules for discretization using Finite Volume Method? b. What are different methods to evaluate matrix inverse for the solution of simultaneous algebraic equations? Compare them. c. ADI scheme

Set No.3 Code No: R05 410306

IV B. Tech I Semester Supplementary Examinations, March 2009 Computational Fluid Dynamics (Mechanical Engineering) Time: 3 hours

Max. Marks: 80

Answer any FIVE questions All Questions carry equal marks ******** 1. Obtain the finite difference formulation for 2-D steady state heat conduction of solid cylinder in r-θ plane subjected to a temperature of TS at its outer surface. Discuss the peculiarities in the discretization of curvilinear grid compared to the Cartesian grid and how the singularities are handled in the present case. 2. A square slab of side, L is subjected to 500C on its right side, 00C on its top side, 1500C on its left side and 2500C on its bottom. Considering the mesh size in x and y directions as L/3, obtain the steady state temperature distribution for the square slab using Guass Seidel Method. What is the temperature distribution at the end of 5 iterations. 3. What is meant by banded matrices? Explain the algorithm to solve tridiagonal matrices. 4. Using the Von Neumann Stability analysis, obtain the stability criterion for the explicit formulation of Parabolic equations. 5. Define Vorticity? How the pressure gradient term is eliminated from the momentum equations using Vorticity – Stream Function method? What are the disadvantages of this method in determination flow field? 6. Explain how pressure and velocity are linked using SIMPLE algorithm for 2D momentum equations? 7. What are different schemes of interpolation used in the discretization of 1D Convection – Diffusion problem? Write about Upwind Schemes in detail and compare them. 8. Write short notes on any two of the following: a. Explain ADI scheme for the discretization of 2D parabolic equations. b. What are different methods to evaluate matrix inverse for the solution of simultaneous algebraic equations? Compare them. c. What are the basic parameters used to assess interpolation schemes in finite volume formulation of Convection – Diffusion problems?

Set No.4 Code No: R05 410306

IV B. Tech I Semester Supplementary Examinations, March 2009 Computational Fluid Dynamics (Mechanical Engineering) Time: 3 hours

Max. Marks: 80

Answer any FIVE questions All Questions carry equal marks ******** 1. List out the properties of Numerical solution methods in detail. 2. Two ends of a fin of cross sectional area 2 cm2, perimeter 2 cm, 100 cm long are maintained at 1270C and 2270C respectively. It losses heat from the surface due to natural convection to the surroundings at 270C with the heat transfer coefficient of 5W/m2K. The thermal conductivity of fin material is 45 W/m K. Find the temperature distribution in the fin taking the mesh size as 20 cm. Compare the finite difference solution with analytical solution and tabulate both the values. 3. a. What are the different types of Numerical grids? Illustrate them by figures. b. What are the relative advantages and disadvantages of Explicit and Implicit methods applied to linear parabolic equations? 4. a. What are the sources of error in the numerical solution of given partial difference equation? b. Derive the expression for the stability criterion for the finite difference solution of the ID unsteady heat conduction using Von Neumann Analysis. 5. A stainless steel rod of 20mm diameter is carrying an electric current of 1000A. The thermal and electrical conductivities are 20 W/mK and 1.25 X 104 (Ω cm)-1. What is the temperature distribution from centre to surface if the outer surface is kept at 4000C taking the mesh size as 2mm. 6. A marble slab ( k = 2 W/mK, α = 1 X 10-6 m2/s ) that is 2 cm thick is initially at a uniform temperature of 2000C. Suddenly, one of its surfaces is lowered to 00C and is maintained at that temperature, while the other surface is kept insulated. Develop an explicit finite difference scheme for the determination of the temperature distribution in the slab at the end of nearly 50 seconds. 7. Define Vorticity? Using Vorticity / Stream function method, derive the expression for momentum equation without pressure term? What are its advantages and disadvantages compared with other methods? 8. Write short notes on any two of the following: a. Convergence techniques in Gauss Seidel method b. Crank Nickolson Scheme c. Turbulence models

Set No. 1

Code No: R05410409

IV B.Tech I Semester Supplimentary Examinations, March 2009 SATELLITE COMMUNICATIONS ( Common to Electronics & Communication Engineering and Electronics & Telematics) Time: 3 hours Max Marks: 80 Answer any FIVE Questions All Questions carry equal marks ⋆⋆⋆⋆⋆ 1. Give Comparative Study between LEO, MEO, GEO and HEO types of Satellites. [16] 2. Discuss in detail the effects of Earth Gravitational Force on Satellite.

[16]

3. Explain how altitude and orbit control is achieved from an earth station.

[16]

4. (a) Why uplink frequency is different from down link frequency in a satellite communication system? Discuss. (b) Derive the general link design equation. Find out the expression for (C/N) and (G/T) ratios. Explain the importance of these ratios in satellite link design. [8+8] 5. A BPSK TDMA system is to transmit 1000 digital voice channels, each with 4 bits per sample at a 64kbps rate. The system must accommodate 1000 data bits/slot at a frame efficiency of 90%. (a) What is the number of slots in a frame? (b) What is the length of TDMA frame? (c) How many preamble bits can be used? (d) What is the required satellite bandwidth?

[16]

6. Explain to why it is necessary to have frequency coordination among earth stations themselves and earth station-terrestrial microwave link? Discuss the techniques to achieve them? [16] 7. In NGSO, give some example of systems explain about each one.

[16]

8. Explain in details about GPS position location principles.

[16]

⋆⋆⋆⋆⋆

1 of 1

Set No. 2

Code No: R05410409

IV B.Tech I Semester Supplimentary Examinations, March 2009 SATELLITE COMMUNICATIONS ( Common to Electronics & Communication Engineering and Electronics & Telematics) Time: 3 hours Max Marks: 80 Answer any FIVE Questions All Questions carry equal marks ⋆⋆⋆⋆⋆ 1. Explain the function of each element of a Mobile Satellite service(MSS).

[16]

2. (a) What are the various orbits in which a satellite can exist? Discuss. (b) Explain how the solar eclipse affect the working of a Communication Satellite. [8+8] 3. (a) List the various satellite subsystems and state their purpose and principal parameter that characterize them quantatively. (b) Explain the following: i. Telemetry ii. Tracking and Command.

[8+4+4]

4. Design a transmitting earth station to provide a clear air C/N of 30dB in a Ku band transponder at a frequency of 14.15GHz. Use a uplink antenna with a diameter of 5 m and an aperture efficiency of 68% and find the uplink transmitter power required to achieve the required C/N. The uplink station is located on the 2 dB contour of the satellite foot print. Allow 1.0dB on the uplink for miscellaneous and clear air losses. [16] 5. (a) Distinguish between bandwidth-limited and power-limited operation as applied to an FDMA network. (b) The EIRP of each earth station in an FDMA network is 47 dBW, and the input data are at the T1 bit rate with 7/8 FEC added. The downlink bit rate is limited to a maximum of 60 Mb/s with 6 dB output back-off applied. Compare the EIRP needed for the earth stations in a TDMA network utilizing the same transponder. What are two components that determine EIRP? [6+10] 6. What is meant by Tracking and Pointing? Explain its significance and the techniques as to how these are achieved? [16] 7. (a) What is meant by Sun syn orbit? (b) Illustrate with example show the Sun syn orbit is active. 8. (a) Explain the following in GPS C\A code accuracy: i. HDOP ii. VDOP 1 of 2

[8+8]

Set No. 2

Code No: R05410409 iii. GDOP.

(b) Explain how the accuracy in GPS measurement is increased? ⋆⋆⋆⋆⋆

2 of 2

[8+8]

Set No. 3

Code No: R05410409

IV B.Tech I Semester Supplimentary Examinations, March 2009 SATELLITE COMMUNICATIONS ( Common to Electronics & Communication Engineering and Electronics & Telematics) Time: 3 hours Max Marks: 80 Answer any FIVE Questions All Questions carry equal marks ⋆⋆⋆⋆⋆ 1. Discuss the Performance Characteristics of different Altitude Satellites.

[16]

2. (a) What is a Geosynchronous Orbit? Discuss the advantages and disadvantages of these orbits. (b) What are Orbital Perturbations? Explain the effects of earth’s oblateness on orbital inclination of Geosynchronous Satellite. [8+8] 3. (a) Explain about 6/4 GHz Communication Subsystem in detail with neat schematics. (b) The earth subtends an angle of 170 when viewed from geostationary orbit. What are the dimensions and gain of horn antenna that will provide global coverage at 4 GHz. [8+8] 4. (a) Explain down link design objectives of a satellite communication system and hence give the down link design of Intel Sat IV-A Satellite. (b) A Satellite carrying an 11.7 GHz signal transmitter is located at 40,000Km from an earth station. The transmitting antenna with gain 17dB radiates a power of 200mW towards earth station. The diameter of the receiving antenna is 10 meters and the aperture efficiency is 50 percent: i. Calculate the receiving antenna gain. ii. The earth station G/T iii. The C/N in a 100 Hz noise bandwidth. If the overall system noise temperature is 12500 K. [8+8] 5. (a) Explain structure of an Intelsat traffic data burst? (b) A TDMA network of five earth stations shares a single transponder equally. The frame duration is 2.0ms, the preamble time per station is 20microsec, and guard band of 5microsec are used between bursts. Transmission bursts are QPSK at 30Mbaud. [6+10] 6. (a) What are the functions of Earth station Tracking system to be performed? (b) Explain the functional elements of a satellite tracking system?

[6+10]

7. (a) What is meant by Sun syn orbit? (b) Illustrate with example show the Sun syn orbit is active.

1 of 2

[8+8]

Set No. 3

Code No: R05410409 8. Explain the principle of operation of GPS in details. ⋆⋆⋆⋆⋆

2 of 2

[16]

Set No. 4

Code No: R05410409

IV B.Tech I Semester Supplimentary Examinations, March 2009 SATELLITE COMMUNICATIONS ( Common to Electronics & Communication Engineering and Electronics & Telematics) Time: 3 hours Max Marks: 80 Answer any FIVE Questions All Questions carry equal marks ⋆⋆⋆⋆⋆ 1. Discuss the applications of Satellite Communication.

[16]

2. (a) The Semi major and Semi minor axis of an Elliptical Satellite Orbit are 20,000 Km and 16,000Km respectively. Determine the apogee and perigee distances. (b) A Geosynchronous Satellite moving in an equatorial circular orbit at a height of 35000 Km above the surface of the Earth gets inclined at an angle of 20 . Calculate the maximum deviation in latitude and also maximum deviation in longitude. Determine the maximum displacements in Km caused by latitude and longitude displacements. [8+8] 3. (a) Discuss the principle of N-S control of a spinner satellite using infrared sensor with a neat figure. (b) Draw the block diagram of typical onboard control system for a spinner satellite and explain its operation. [8+8] 4. (a) Derive the equation for the power received by an earth station from a satellite transmitter. (b) A Satellite located in geosynchronous orbit 38,000Km from an earth station ,operates at a frequency of 4 GHz and radiates a power of 5W from an antenna with a gain of 10dB in the direction of the earth station. Find the received power, if the receiving antenna has a gain of 52.3 dB. [8+8] 5. (a) Define Bits, Symbols and channels in TDMA. (b) Explain TDMA frame structure.

[3+13]

6. Explain to why it is necessary to have frequency coordination among earth stations themselves and earth station-terrestrial microwave link? Discuss the techniques to achieve them? [16] 7. Explain the following: (a) Traffic distribution and coverage (b) Satellite capacity (c) State of spacecraft.

[8+4+4]

8. Explain about the GPS receiver operation. ⋆⋆⋆⋆⋆ 1 of 1

[16]

Set No.1 Code No: R05 410503

IV B. Tech I Semester Supplementary Examinations, March 2009 Data Ware Housing & Data Mining (Computer Science Engineering) Time: 3 hours

Max. Marks: 80 Answer any FIVE questions All Questions carry equal marks ********

1. a) What is data mining? Briefly explain the Knowledge discovery process. b) Explain the three-tier data warehouse architecture. 2. a) Briefly describe various forms of data pre-processing. b) What is data cleaning? Describe the approaches to fill missing values. 3. Briefly describe the primitives for specifying a data mining task. 4. What are the quantitative association rules? What is ARCS and discuss the involved steps. 5. a) Describe the criteria used to evaluate classification and prediction methods. b) With an example, explain the classification by decision tree induction. 6. a) What is Density based clustering? Describe DBSCAN clustering algorithm. b) What is Grid based clustering? Describe any one Grid based clustering algorithm. 7. Briefly discuss about document classification and document cluster analysis. 8. What is conceptual clustering? Describe about COBWEB.

Set No.2 Code No: R05 410503

IV B. Tech I Semester Supplementary Examinations, March 2009 Data Ware Housing & Data Mining (Computer Science Engineering) Time: 3 hours

Max. Marks: 80 Answer any FIVE questions All Questions carry equal marks ********

1. a) What is a concept hierarchy? Describe the OLAP operations in the Multidimensional data model. b) What is association and correlation? With an example describe classification and prediction. 2. a) What is data dispersion? Describe the common measures for data dispersion. b) What is data integration? Discuss the issues to be considered for data integration. 3. Discuss the concept involved in designing GUI based on DMQL. 4. What is constraint-based mining? Discuss about the possible constraints in high-level declarative DMQL and user interface. 5. a) What is attribute selection measure? Briefly describe the attribute selection measures for decision tree induction. b) With an example, explain the classification by decision tree induction. 6. a) What is partitioning method? Describe any one partition based clustering algorithm. b) What is Hierarchical clustering? Describe any one Hierarchical clustering algorithm. 7. What is text mining? Describe about basic measures for text retrieval. 8. What are the quantitative association rules? What is ARCS, discuss the involved steps in ARCS.

Set No.3 Code No: R05 410503

IV B. Tech I Semester Supplementary Examinations, March 2009 Data Ware Housing & Data Mining (Computer Science Engineering) Time: 3 hours

Max. Marks: 80 Answer any FIVE questions All Questions carry equal marks ********.

1. a) Draw and explain the architecture of a typical data mining system. b) Why data mining functionalities are used? Explain with an example data characterization and data discrimination. 2. a) What is noisy data? Explain the binning methods for data smoothening. b) What is data reduction? Discuss about dimensionality reduction. 3 Discuss the available techniques for concept Hierarchy Generation for categorical data. 4. a) What is a data ware house? Differentiate between operational data base system and data warehouses. b) Describe the procedure for mining class comparisons. 5. Discuss about multidimensional association rule mining from relational databases. 6. a) What are Bayesian classifiers? With an example, describe how to predict a class label using naive Bayesian classification. b) What is back propagation? Describe back propagation algorithm. 7. a) What is cluster analysis? Describe the dissimilarity measures for categorical, ordinal and ratio-scaled variables. b) Describe how categorization of Major Clustering methods is being done. 8. What is spatial data mining? What is spatial data cube, and what are the three dimensions in a spatial data cube?

Set No.4 Code No: R05 410503

IV B. Tech I Semester Supplementary Examinations, March 2009 Data Ware Housing & Data Mining (Computer Science Engineering) Time: 3 hours

Max. Marks: 80 Answer any FIVE questions All Questions carry equal marks ********

1. a) Describe the three challenges to data mining regarding data mining methodology and user interaction issues. b) With an example, describe snowflake and fact constellations. 2. a) What is the use of data discretization? Explain entropy based data discretization? b) What is data normalization? Explain any two normalization methods. 3. Describe about data generalization, summarization and analytical characterization. 4. What is transactional database? With an example, explain multilevel association rule mining. 5. a) What is backpropogation? Describe backpropogation algorithm. b) Briefly outline the major steps of decision tree classification. 6. a) What is cluster analysis? Describe the dissimilarity measures for intervalscaled variables and binary variables. b) Describe how categorization of major clustering methods is being done. 7. What is multimedia data mining? What kind of associations can be mined in multimedia data? 8. With an example, explain a) any one partition based clustering algorithm. b) any one hierarchical clustering algorithm.

Set No. 1

Code No: R05410805

IV B.Tech I Semester Supplimentary Examinations, March 2009 PROCESS MODELING AND SIMULATION (Chemical Engineering) Time: 3 hours Max Marks: 80 Answer any FIVE Questions All Questions carry equal marks ⋆⋆⋆⋆⋆ 1. Explain the convergence procedure to find the bubble point temperature of a binary vapour-liquid mixture. [16] 2. An irreversible exothermic reaction is carried out in a single perfectly mixed nonisothermal CSTR. The reaction is A→ B The reaction is nth order in reactant A and has heat of react λ (energy units/mole of A reacted). Negligible heat losses and constant densities are assumed. To remove the heat of reaction, a cooling jacket surrounds the reactor. Cooling water is added to the jacket at a constant volumetric flow rate. Develop a mathematical model for the system assuming that the CSTR has a perfectly mixed cooling jacket. State all the assumptions made and explain the notations scheme used clearly. [16] 3. Write the model equations involved in a Batch reactor when it is in the heating phase. [16] 4. Solve un+2 - 4un+1 + 4un = 2n .

[16]

5. Solve: yx+1 − yx + xyx+1 yx = 0 given y1 = 5.

[16]

6. Butyl acetate is to be produced in a battery of continuous stirred tank reactors operating in series by reacting 2000 kg/h of butanol with 326 kg/h of glacial acetic acid containing sufficient sulphuric acid to catalyze the reaction at 100 C. Under these conditions the rate of reaction can be expressed by the equation : r = k CA2 . where k is 0.28 m/kg mole.min and CA is the concentration of acetic acid in kg moles/m3 . If the effective volume of each tank is 10 m3 and the density of the reaction mixture is assumed to be constant 280 kg/m3 , estimate the number of reaction vessels required if the concentration of the the acetic acid in the final discharge is not to exceed 18 kg/ m3 . [16] 7. Derive the mass balance equations for interacting and non-interacting tanks and also write the computer program for simulation. [16] 8. Write short notes on the following: (a) Indcidence matrix (b) Adjacency matrix.

[8+8] ⋆⋆⋆⋆⋆

1 of 1

Set No. 2

Code No: R05410805

IV B.Tech I Semester Supplimentary Examinations, March 2009 PROCESS MODELING AND SIMULATION (Chemical Engineering) Time: 3 hours Max Marks: 80 Answer any FIVE Questions All Questions carry equal marks ⋆⋆⋆⋆⋆ 1. (a) What is continuity equation, discuss it. (b) Consider the tank of perfectly mixed liquid into which flows a liquid stream at a volumetric rate of Fo and with a density of ρ0 . The volumetric flow rate from the tank is F, and the density of the out flowing stream is the same as that of the tank’s contents. Write the total continuity equation. [6+10] 2. Consider a two perfectly mixed tanks in series, the flow rate of oil passing through the tanks is F, density ρ heat capacity Cp . The volume of 1st tank is V1 and 2nd tank is V2 , which are constant. The temperature of oil entering 1st tank is To . The temperatures in the two tanks are T1 & T2 . A heating coil in the 1st tank uses steam to heat the oil. Write the energy balances for both the tanks. [16] 3. (a) Differentiate between “Mass-transfer Limited” and “Chemical-rate Limited reactors. (b) Derive the model equations involved in the “reactor with mass transfer”. [6+10] 4. Solve up+2 - 6up+1 + 9 up =0.

[16]

5. A continuous flow distillation column is fed with a binary mixture of A and B. The relative volatility (α) of mixture is constant. Consider the plates above the feed plate. Derive the difference equations which relates the liquid composition to the plate number n, if the overall plate efficiency is 100% and explain the method of solving it. [16] 6. Solve the following: y ′ (t) − y (t − 1) = t, if y(t)=0 for t ≤ 0.

[16]

7. Write down a computer simulation program to carry out dew point calculations. [16] 8. Write short notes on the following: (a) Indcidence matrix (b) Adjacency matrix.

[8+8] ⋆⋆⋆⋆⋆

1 of 1

Set No. 3

Code No: R05410805

IV B.Tech I Semester Supplimentary Examinations, March 2009 PROCESS MODELING AND SIMULATION (Chemical Engineering) Time: 3 hours Max Marks: 80 Answer any FIVE Questions All Questions carry equal marks ⋆⋆⋆⋆⋆ 1. Consider a CSTR in which consecutive 1st order reactions A′′ → B → C occur k1 k2 with exothermic heats of reaction A −→ B −→ C with exothermic heats of reaction λ1

λ2

λ1 and λ2 . Write the energy equation.

[16]

2. An ice cube is dropped into a hot, perfectly mixed, insulated cup of coffee. List all assumption and define all terms. Develop the equations describing the dynamics of the system. [16] 3. Explain the Mass Transfer Phenomena in a Reactor with a case study.

[16]

2 4. Solve yx+1 − 3yx+1 yx + 2y 2x = 0

[16]

5. Calculate the number of plates required to absorb 99 % mole ketene from 135 kgmoles/h of gas containing 4.36% ketene by volume using 138 kgmoles/h of glacial acetic acid. The plate efficiency, of a 3.5 m diameter tower containing 0.7D weirs and therefore a plate hold-up of 2.09 kgmoles liquid is 40%. The pseudo first order reaction rate constant for the chemical absorption process may be taken to be 0.75 sec−1 and the equilibrium relationship Y* = 2 X Where Y* is the kgmoles ketene kg mole of carrier gas in equilibrium with X kg moles ketene / kg of acetic acid in the liquid. [16] 6. Solve the following: y ′ (t) − y (t − 1) = t, if y(t)=0 for t ≤ 0.

[16]

7. Write down the mass balance equation for interacting system and simulate it using R-k fourth order method. [16] 8. What is Precedence ordering of equation sets and explain how it is done through an example. [16] ⋆⋆⋆⋆⋆

1 of 1

Set No. 4

Code No: R05410805

IV B.Tech I Semester Supplimentary Examinations, March 2009 PROCESS MODELING AND SIMULATION (Chemical Engineering) Time: 3 hours Max Marks: 80 Answer any FIVE Questions All Questions carry equal marks ⋆⋆⋆⋆⋆ 1. The liquid phase hydrolysis reaction of acetic anhydride to form acetic acid is carried out in a constant volume adiabatic batch reactor. The reaction is exothermic with the following stoichiometry. (CH3 C)2 O+H2 O → 2CH3 COOH+Heat. Derive mass and energy balances for the system assuming the reaction as first order. [16] 2. An irreversible exothermic reaction is carried out in a single perfectly mixed nonisothermal CSTR. The reaction is A→ B. The reaction is nth order in reactant A and has heat of react λ (energy units/mole of A reacted). Negligible heat losses and constant densities are assumed. To remove the heat of reaction, a cooling jacket surrounds the reactor. Cooling water is added to the jacket at a constant volumetric flow rate. Develop a mathematical model for the system assuming that the CSTR has a plug flow cooling jacket. State all the assumptions made and explain the notations scheme used clearly. [16] 3. (a) Describe the ideal binary distillation column with a neat sketch. (b) Write the Model equations for “Condenser and Reflux drum” in ideal binary distillation column. [8+8] 4. Solve un+2 - 4un+1 + 4un = 2n .

[16]

5. Solve the difference equation Yn+2 Yn = Yn+2 2 .

[16]

6. 8000 kg/h of a 5% by weight solution of acetaldehyde in toluene is to be treated with 5000 kg/h of water in a 6 theoretical stage extraction column operating under counter flow conditions. If the concentration of the feed conditions is suddenly changed to a 3% by weight solution, how long will it take for the extractor to settle down to steady state operation under the new condition? The equilibrium relation is Y = 2.2 X Where, Y = Kg of Acetaldehyde per Kg of water X = Kg of Acetaldehyde per Kg of toluene, The holdup per stage can be assumed to be constant at 750 Kg toluene; 300 Kg water. [16] 7. A stirred tank reactor of effective volume, V m3 containing a solution of a reactant A, of concentration CAo is heated to such a temperature that the pseudo first order reaction : A → B takes place. When the chemical reaction is initiated a solution of A of concentration CAi is fed continuously to the reactor at the rate of q m3 /min. If the specific reaction rate is k min−1 and the vessel is assumed to contain a perfect 1 of 2

Set No. 4

Code No: R05410805

stirrer, develop an expression giving the concentration of A in the effluent as a function of time. [16] 8. Explain about Simultaneous modular approach with a neat flow chart in process plant simulation. [16] ⋆⋆⋆⋆⋆

2 of 2

Set No. 1

Code No: R05411204

IV B.Tech I Semester Supplimentary Examinations, March 2009 DISTRIBUTED DATA BASES ( Common to Information Technology and Computer Science & Systems Engineering) Time: 3 hours Max Marks: 80 Answer any FIVE Questions All Questions carry equal marks ⋆⋆⋆⋆⋆ 1. (a) Write about horizontal and vertical fragmentation. (b) Given a global relation EMP(EMPNUM,NAME,SAL,TAX,MGRNUM,DEPTNUM) Write the mixed fragmentation definition and fragmentation tree of relation EMP. [8+8] 2. (a) For given EMP and DEPT relations, assuming the necessary attributes a query to give the names of employees who work in a department whose manager has number 373 but who do not earn more than Rs.100000/- is as given below. Simplify the query explaining the steps involved. PJ EMP.NAME((EMP JN DEPTNUM=DEPTNUM SL MGRNUM=373DEP T ) DF (SL SAL> 100000 EMP JN DEPTNUM=DEPTNUMSL MGRNUM=373DEP T ) (b) Discuss query processing in detail with an example.

[8+8]

3. Discuss the following for distributed databases (a) Post-optimization (b) Transmission requirements (c) AHY approach for semi-Joins.

[5+5+6]

4. (a) Discuss the architectural aspects of distributed transactions (b) Explain the following: i. Multiple copies of the data ii. Transaction failures.

[8+8]

5. What is Concurrency Control and explain about Serializability in a Centralized Database? [16] 6. Explain in detail about the following: (a) Primary site approach (b) Checkpoints (c) Cold Restart.

[6+6+4]

7. Write a short note on the following: 1 of 2

Set No. 1

Code No: R05411204 (a) Client buffer management (b) Server buffer management (c) Characteristics of object models.

[6+4+6]

8. Explain the following in detail: (a) Object Management Architecture (b) COM/OLE and Database Interoperability. ⋆⋆⋆⋆⋆

2 of 2

[10+6]

Set No. 2

Code No: R05411204

IV B.Tech I Semester Supplimentary Examinations, March 2009 DISTRIBUTED DATA BASES ( Common to Information Technology and Computer Science & Systems Engineering) Time: 3 hours Max Marks: 80 Answer any FIVE Questions All Questions carry equal marks ⋆⋆⋆⋆⋆ 1. Consider the global relations: PATIENT(NUMBER, NAME, SSN, AMOUNT-DUE,DEPT,DOCTOR,MED-TREATMENT) DEPARTMENT(DEPT,LOCATION,DIRECTOR) STAFF(STAFFNUM,DIRECTOR,TASK) Define their fragmentation as follows: (a) DEPARTMENT has a horizontal fragmentation by LOCATION, with two locations; each department is conducted by one DIRECTOR. (b) There are several staff members for each department, led by the departments director. STAFF has a horizontal fragmentation derived from that of the DEPARTMENT and a semi-join on the DIRECTOR attribute. Which assumption is required in order to assure completeness? (c) Discuss the type of fragmentation PATIENT can have. [16] 2. (a) Prove that the following two queries have the same semantics PJ NAME,DEPTNUMSL DEPTNUM=15EM P SL DEPTNUM=15P J NAME,DEPTNUMEM P (b) Draw an operator tree for the following query PJ SNUMSL AREA=“North”(SUPPLY JN DEPTNUM=DEPTNUMDEP T )[8+8] 3. (a) Discuss the problems in Query Optimization. (b) Discuss how query optimisation is useful in distributed databases.

[8+8]

4. (a) Discuss the reference model of distribution transaction recovery. (b) Write about granularity based locking.

[8+8]

5. (a) Explain preemptive and nonpreamptive methods? (b) Explain Distributed deadlock detection algorithm?

[8+8]

6. Write the local recovery procedure for 3-phase-commitment.

[16]

7. Describe in detail cash consistency and object identifier management in object management? [16] 8. Explain the Push-Based Technologies in detail. ⋆⋆⋆⋆⋆ 1 of 1

[16]

Set No. 3

Code No: R05411204

IV B.Tech I Semester Supplimentary Examinations, March 2009 DISTRIBUTED DATA BASES ( Common to Information Technology and Computer Science & Systems Engineering) Time: 3 hours Max Marks: 80 Answer any FIVE Questions All Questions carry equal marks ⋆⋆⋆⋆⋆ 1. (a) Write about different types of fragmentation in detail. (b) Explain with an example to construct global relations from fragments. [8+8] 2. (a) Draw an operator tree and show the simplification for the following query. For given EMP and DEPT relations, assuming the necessary attributes, a query to give the names of employees who work in a department whose manager has number 373 but who do not earn more than Rs.100000/- is as given below. PJ EMP.NAME((EMP JN DEPTNUM=DEPTNUM SL MGRNUM=373DEP T ) DF (SL SAL>100000 EMP JN DEPTNUM=DEPTNUMSL MGRNUM=373DEP T ) (b) Explain parametric queries in detail.

[8+8]

3. (a) Discuss the problems in Query Optimization. (b) Discuss how query optimisation is useful in distributed databases.

[8+8]

4. (a) Write about the atomicity of transactions in distributed databases with emphasis on failures and logs and recovery methods. (b) Explain the 2-phase commitment protocol.

[8+8]

5. (a) Explain false deadlocks in detail? (b) Explain Deadlock prevention in distributed databases? (c) Explain the differences between preemptive and non preemptive methods? [4+6+6] 6. Explain the various problems which arise when trying to design a reliable distributed database system? [16] 7. Write a short note on: (a) Synchronizing access to objects (b) Management of Type lattice. (c) Explain about transaction models and object structures. 8. Explain the following in detail: (a) Datawarehousing architectures

1 of 2

[5+5+6]

Set No. 3

Code No: R05411204 (b) OLAP Datamodel (c) OLAP Servers.

[4+6+6] ⋆⋆⋆⋆⋆

2 of 2

Set No. 4

Code No: R05411204

IV B.Tech I Semester Supplimentary Examinations, March 2009 DISTRIBUTED DATA BASES ( Common to Information Technology and Computer Science & Systems Engineering) Time: 3 hours Max Marks: 80 Answer any FIVE Questions All Questions carry equal marks ⋆⋆⋆⋆⋆ 1. (a) Consider the global relations: PATIENT(NUMBER, NAME, SSN, AMOUNT-DUE,DEPT,DOCTOR,MEDTREATMENT) DEPARTMENT(DEPT,LOCATION,DIRECTOR) STAFF(STAFFNUM,DIRECTOR,TASK) Define their fragmentation as follows: i. DEPARTMENT has a horizontal fragmentation by LOCATION, with two locations; each department is conducted by one DIRECTOR. ii. There are several staff’s members for each department, led by the department’s director. STAFF has a horizontal fragmentation derived from that of the DEPARTMENT and a semi-join on the DIRECTOR attribute. Which assumption is required in order to assure completeness? (b) Discuss the levels of distribution transparency in brief.

[8+8]

2. (a) Prove that the following two queries have the same semantics PJ NAME,DEPTNUMSL DEPTNUM=15EM P SL DEPTNUM=15P J NAME,DEPTNUMEM P (b) Draw an operator tree for the following query PJ SNUMSL AREA=“North”(SUPPLY JN DEPTNUM=DEPTNUMDEP T )[8+8] 3. (a) Explain the use of semi-Join for Join quarics with example. (b) Discuss how query optimization is useful in distributed databases.

[8+8]

4. (a) Write about concurrency control based on locking in centralised databases. (b) Write about the concurrency control based on locking in distributed databases. [8+8] 5. (a) Explain the conservative timestamp method. (b) Consider the data item x. Let RTM(x)=25 and WTM(x)=20. Let the pair(Ri(x), TS) (Wi(x), TS) denote the read(write) request of transaction Ti on the item x with timestamp TS. Indicate the behavior of the basic timestamp method with the following sequence of requests. (R1(x), 19), (R2(x), 22), (w3(x), 21) (W4(x), 23), (R5(x), 28), (W6(x), 27). [6+10]

1 of 2

Set No. 4

Code No: R05411204 6. Write a short note on the following: (a) Content of Catalogs (b) Centralized catalogs (c) Fully replicated catalogs (d) Local catalogs.

[8+2+2+4]

7. Write a short note on the following: (a) Object identifier management (b) Object clustering (c) Object migration.

[5+6+5]

8. Explain the following : (a) Delivery schedule generation (b) Propagating updates.

[8+8] ⋆⋆⋆⋆⋆

2 of 2

Set No. 1

Code No: R05411205

IV B.Tech I Semester Supplimentary Examinations, March 2009 ADVANCED COMPUTING CONCEPTS ( Common to Information Technology and Computer Science & Systems Engineering) Time: 3 hours Max Marks: 80 Answer any FIVE Questions All Questions carry equal marks ⋆⋆⋆⋆⋆ 1. Describe the basic concepts and technical components of Grid Computing?

[16]

2. Discuss the five levels of autonomic computing implementation?

[16]

3. What is meant by system tuning? What factors are focused in tuning a system? [16] 4. (a) What is ALAP start time? (b) Write the procedure for computing start time with ALAP. (c) Write the insertion scheduling heuristic algorithm 5. Define the following with respect to NanOS. a) Object b) Agent c) Kernel Agent d) User agent e) Task 6. What is the role of the internet in pervasive computing? Explain.

[4+8+4]

[16] [16]

7. (a) What is meant by Synchronization? Write the various Synchronization Scenarios? (b) Give an example for Synchronization?

[8+8]

8. Draw the copying circuit. Why is it impossible to copy an unknown state? Explain. [16] ⋆⋆⋆⋆⋆

1 of 1

Set No. 2

Code No: R05411205

IV B.Tech I Semester Supplimentary Examinations, March 2009 ADVANCED COMPUTING CONCEPTS ( Common to Information Technology and Computer Science & Systems Engineering) Time: 3 hours Max Marks: 80 Answer any FIVE Questions All Questions carry equal marks ⋆⋆⋆⋆⋆ 1. Describe the basic concepts and technical components of Grid Computing?

[16]

2. (a) What is a control loop? Draw the diagram of control loop? (b) What are the elements of basic sub elements of a control loop?

[8+8]

3. (a) Explain tuning of caching strategies and operating systems. (b) Why are I/O subsystems used in conventional servers are not good for Cluster nodes? Explain. [8+8] 4. (a) Write the difference of scheduling in distributed homogeneous and heterogeneous system. (b) What are the strategies of scheduling in clustering? 5. In what way Beowulf are scaled to 100 processors.Explain.

[10+6] [16]

6. (a) How does pervasive computing has become a new dimension of personal computing. Explain? (b) Explain any one pervasive computing scenario.

[8+8]

7. (a) What are the steps involved in a typical WAP email session? (b) Show how an OSGi gateway is integrated into a car information system.[8+8] 8. What is meant by quantum teleportation? Explain. ⋆⋆⋆⋆⋆

1 of 1

[16]

Set No. 3

Code No: R05411205

IV B.Tech I Semester Supplimentary Examinations, March 2009 ADVANCED COMPUTING CONCEPTS ( Common to Information Technology and Computer Science & Systems Engineering) Time: 3 hours Max Marks: 80 Answer any FIVE Questions All Questions carry equal marks ⋆⋆⋆⋆⋆ 1. Write briefly about Globus Toolkit tools and APIs.

[16]

2. From where does autonomic computing is derived? How does autonomic computing meet the challenges of IT? [16] 3. (a) What is the first step in developing a cluster? Explain (b) Write the advantages of front end in cluster? (c) How can all nodes in cluster can be installed at a time? Is it possible.hfill[8+4+4] 4. (a) What is ALAP start time? (b) Write the procedure for computing start time with ALAP. (c) Write the insertion scheduling heuristic algorithm 5. Explain the components of a PC based SMP cluster.

[4+8+4] [16]

6. (a) Explain about Haptic interfaces (b) Explain Tegic T9 technology.

[8+8]

7. (a) Write the features of Suns KVM (b) Mention some of class Libraries of IBMs visual Age Micro edition.

[8+8]

8. How is bounded error, quantum polynomial time related other complexity classes. Give examples of algorithm that run in BQP time. [16] ⋆⋆⋆⋆⋆

1 of 1

Set No. 4

Code No: R05411205

IV B.Tech I Semester Supplimentary Examinations, March 2009 ADVANCED COMPUTING CONCEPTS ( Common to Information Technology and Computer Science & Systems Engineering) Time: 3 hours Max Marks: 80 Answer any FIVE Questions All Questions carry equal marks ⋆⋆⋆⋆⋆ 1. Illustrate the Grid Architecture with a diagram.

[16]

2. Explain IBM’s DB2 DBMS.

[16]

3. (a) Explain the switching techniques of cluaster interconnects. (b) What are the interconnection devices used with Fast Ethernet. Explain with a diagram. [8+8] 4. What is the goal of dynamic load balancing? Mention the load balancing strategies. Compare them. [16] 5. (a) what is meant by reliable object Invocation ? for what purposes it is used? (b) Write about HIDRA concurrency control.

[8+8]

6. (a) How does pervasive computing has become a new dimension of personal computing. Explain? (b) Explain any one pervasive computing scenario.

[8+8]

7. (a) What steps does Synchronization Protocols Consists of? (b) What are the Characteristics of Jini? 8. Write about quantum circuits.

[8+8] [16]

⋆⋆⋆⋆⋆

1 of 1

Set No.1 Code No: R05 411206

IV B. Tech I Semester Supplementary Examinations, March 2009 IMAGE PROCESSING (Information Technology) Time: 3 hours

Max. Marks: 80

Answer any FIVE questions All Questions carry equal marks ******** 1. Define an image. What are the fundamental steps in digital image processing? Explain each of them in detail 2. Consider the two image subsets, s1 and s2, shown in the following figure. For V={1}, determine whether these two subsets are (i) 4-adjacent (ii) 8-adjacent or (iii) m-adjacent. S2 0 0 1 0 0

S1 0 1 1 0 0

0 0 0 0 0

0 0 0 1 1

0 1 1 1 1

0 0 0 1 1

0 1 1 0 1

1 0 0 0 1

10 01 00 00 11

3. a) Develop a procedure for computing the median of an nxn neighborhood. b) Write a technique for updating the median as the centre of the neighborhood is moved from pixel to pixel 4. a) Explain the minimum mean square error filtering with suitable example. b) Explain in detail about the inverse filtering. 5. What are the color models in color image processing? Explain each of them in detail. 6. What are the image compression models? Explain each of them with suitable example. 7. Distinguish between spatial domain techniques and frequency domain techniques of image enhancement. 8. Write short notes on the following (i)

Thresholding

(ii)

Color image processing

Set No.2 Code No: R05 411206

IV B. Tech I Semester Supplementary Examinations, March 2009 IMAGE PROCESSING (Information Technology) Time: 3 hours

Max. Marks: 80

Answer any FIVE questions All Questions carry equal marks ******** 1. What is digital image processing ? What are the components of an image processing system ? Explain each of them in detail. 2. Explain the simple image formation model and discuss about the relationships between pixels. 3. (a) Develop a procedure for computing the median of an n x n neighborhood. (b) Write a technique for updating the median as the centre of the neighborhood is moved from pixel to pixel. 4. A certain x-ray imaging geometry produces a blurring degradation that can be model as the convolution of the sensed image with the spatial, circularly symmetric function.

Where r2 = x2 + y2. Show that the degradation in the frequency domain is given by the expression H(µ 1 v ) =

5. (a)Distinguish between the CMY and CMYK color models. (b)Explain in detail about the histogram processing. 6. (a)Draw the relevant diagram for source encoder and source decoder. 7. (a) What is the limiting effect of repeatedly dilating an image ? Assume that a trivial structuring element is not used. (b).Write any four applications of gray – scale morphology. 8. Write short notes on the following. (i) Thresholding (ii)Error – free compression.

Set No.3 Code No: R05 411206

IV B. Tech I Semester Supplementary Examinations, March 2009 IMAGE PROCESSING (Information Technology) Time: 3 hours

Max. Marks: 80

Answer any FIVE questions All Questions carry equal marks ******** 1. What is digital processing? What are components of an image processing system? Explain each of them in detail. 2. Consider the two image subsets, s1 and s2, shown in the following figure. For V={1}, determine whether these two subsets are (i) 4-adjacent (ii) 8-adjacent or (iii) m-adjacent. S2 S1 0 1 1 0 0

0 0 0 0 0

0 0 0 1 1

0 1 1 1 1

0 0 0 1 1

0 0 1 0 0

0 1 1 0 1

1 0 0 0 1

10 01 00 00 11

3. Suppose that a digital image is subjected to histogram equalization. Show that a second pass of histogram equalization will produce exactly the same result as the first pass.

4. A certain x-ray imaging geometry produces a blurring degradation that can be model as the convolution of the sensed image with the spatial, circularly symmetric function.

5. 6. 7.

8.

Where r2 = x2 + y2. Show that the degradation in the frequency domain is given by the expression H(µ 1 v ) = What are the color models in color image processing? Explain each of them in detail. What are the image compression models? Explain each of them with suitable example. a) What is the limiting effect of repeatedly dilating an image? Assume that a trivial structuring element is not used. b) Write any four applications of gray-scale morphology. Write short notes on the following (i) Image sampling and Quantization (ii) Gray-Level transformation.

Set No.4 Code No: R05 411206

IV B. Tech I Semester Supplementary Examinations, March 2009 IMAGE PROCESSING (Information Technology) Time: 3 hours

Max. Marks: 80

Answer any FIVE questions All Questions carry equal marks ******** 1. Define an image. What are the fundamental steps in digital image processing? Explain each of them in detail. 2. Explain the simple image formation model and discuss about relationships between pixels. 3. Suppose that a digital image is subjected to histogram equalization. Show that a second pass of histogram equalization will produce exactly, the same result as the first pass. 4. a) Explain the minimum mean square error filtering with suitable example. b) Explain in detail about the inverse filtering. 5. a) Distinguish between the CMY and CMYK color models. b) Explain in detail about the histogram processing. 6. a) Draw and Explain a general compression system model. b) Draw the relevant diagram for source encoder and source decoder. 7. Distinguish between spatial domain techniques and frequency domain techniques of image enhancement. 8. Write short notes on the following i) Image segmentation ii) histogram processing

Set No. 1

Code No: R05412305

IV B.Tech I Semester Supplimentary Examinations, March 2009 BIOTECHNOLOGY FOR CROP IMPROVEMENT (Bio-Technology) Time: 3 hours Max Marks: 80 Answer any FIVE Questions All Questions carry equal marks ⋆⋆⋆⋆⋆ 1. Differentiate between:(a) Modifying genes and polygenes. (b) Pleiotrophy and linkage (c) Expressivity and penetrance.

[16]

2. Discuss various pathways of haploid production emphasizing its merits and demerits. [16] 3. How transgenic plants can be produced by manipulating photosynthesis, discuss different approaches. [16] 4. Describe about the strategies employed for genetically engineered plants for herbicide resistance in crop plants. [16] 5. What is expression profiling? Discuss the effect of environmental perturbation on gene expression. [16] 6. Write a detail note on simple sequence repeat polymorphism (SSRP).

[16]

7. How can one increase the frequency with which transposons insert with in the gene of interest?. [16] 8. Describe how pyramiding of major genes can be accomplished using MAS for durable resistance. [16] ⋆⋆⋆⋆⋆

1 of 1

Set No. 2

Code No: R05412305

IV B.Tech I Semester Supplimentary Examinations, March 2009 BIOTECHNOLOGY FOR CROP IMPROVEMENT (Bio-Technology) Time: 3 hours Max Marks: 80 Answer any FIVE Questions All Questions carry equal marks ⋆⋆⋆⋆⋆ 1. What do you understand by variations? Discuss in detail about quantitative and qualitatively inherited variations. [16] 2. What are cybrids. How cybrids can be developed by protoplast fusion and how they are different from somatic hybrids. [16] 3. What is oxidative stress. Discuss its adverse effect on plant. Write how the protection of plants can be accomplished by transgenic approach. [16] 4. Discuss the strategy for production of herbicide resistant plants with relevant examples. [16] 5. What is golden rice? How it is developed by transgenic approach and explain its benefits? [16] 6. Write short notes on the following: (a) Dominant molecular markers (b) Co-dominant molecular markers. 7. Explain in detail about the gene tagging by endogenous transposons.

[8+8] [16]

8. Discuss the essential requirements for marker assisted selection in plant breeding. [16] ⋆⋆⋆⋆⋆

1 of 1

Set No. 3

Code No: R05412305

IV B.Tech I Semester Supplimentary Examinations, March 2009 BIOTECHNOLOGY FOR CROP IMPROVEMENT (Bio-Technology) Time: 3 hours Max Marks: 80 Answer any FIVE Questions All Questions carry equal marks ⋆⋆⋆⋆⋆ 1. Write short note on the following: (a) Genotype and phenotype (b) Qualitative and Quantitative characters (c) Pleiotropy (d) Penetrance & Expressivity.

[16]

2. Give an account of the anther culture.

[16]

3. What are LEA proteins? Explain how LEA proteins can be used for conferring resistance to salt stressed water deficit. [16] 4. Write about the production of bromoxynil and sulphonyl urea resistant transgenic plants. [16] 5. Explain the mechanism of inhibition of ethylene in transgenic plants using anti sense RNA genes during tomato ripening. [16] 6. Give a detail account of cleaved amplified polymorphic sequences (CAPS) and its applications. [16] 7. What is targeted gene? Explain various strategies in gene tagging.

[16]

8. Explain the nearly isogenic line (NIL) strategy for determining amplified loci of targeted gene. [16] ⋆⋆⋆⋆⋆

1 of 1

Set No. 4

Code No: R05412305

IV B.Tech I Semester Supplimentary Examinations, March 2009 BIOTECHNOLOGY FOR CROP IMPROVEMENT (Bio-Technology) Time: 3 hours Max Marks: 80 Answer any FIVE Questions All Questions carry equal marks ⋆⋆⋆⋆⋆ 1. Write short notes on the following: (a) Threshold characters (b) Modifying genes (c) Gene interaction (d) Linkage.

[16]

2. Write short notes on the following: (a) Protoclonal variations (b) Gametoclonal variations (c) Colchiploids (d) Haploids.

[4×4]

3. Discuss in detail whether it is possible to genetically engineer plants to fix nitrogen. [16] 4. What is interference RNA, how it is used for production of virus resistant plants. [16] 5. What are poly fructans? How fructans quality can be improved in plants using bacterial Sac B gene. [16] 6. Write in detail about random amplified hybridization micro satellite and its uses in general . [16] 7. Give an account of transposable elements in prokaryotes.

[16]

8. What are morphological markers? How they produce from the mutagenesis. [16] ⋆⋆⋆⋆⋆

1 of 1

Set No. 1

Code No: R05412306

IV B.Tech I Semester Supplimentary Examinations, March 2009 CANCER BIOLOGY (Bio-Technology) Time: 3 hours Max Marks: 80 Answer any FIVE Questions All Questions carry equal marks ⋆⋆⋆⋆⋆ 1. Explain: (a) EGF (b) TGFα (c) TNF (d) HGF.

[16]

2. Give a detailed account of structure of p53 gene and mutation in human tumours? [16] 3. What are prostaglandins? Explain the prostaglandin synthase-induced co-oxidation? [16] 4. What are oncogenes? Explain any two oncogenes in radiation carcinogenesis and their action? [16] 5. Write short notes on: (a) Angiogenesis. (b) Tumor viruses and cancer.

[8+8]

6. Explain: (a) Role of lytic/(lysis) enzymes in metastasis cascade. (b) Role plasma membrane components in metastasis.

[8+8]

7. Write about the Cancer classification by expression profiling.

[16]

8. Give an account on the cellular sites of traditional cytotoxic drugs.

[16]

⋆⋆⋆⋆⋆

1 of 1

Set No. 2

Code No: R05412306

IV B.Tech I Semester Supplimentary Examinations, March 2009 CANCER BIOLOGY (Bio-Technology) Time: 3 hours Max Marks: 80 Answer any FIVE Questions All Questions carry equal marks ⋆⋆⋆⋆⋆ 1. What are the different diagnostic aspects of identifying tumors? Explain them? [16] 2. Explain tumours of : (a) Reticuloendothelial system (b) Tumours of nervous sytem.

[8+8]

3. Explain the process of oncogene activation and inactivation of tumour suppressor genes by alkylating agents? [16] 4. What is signal transduction? Explain the effects on cell-cell signaling during the radiation carcinogenesis? [16] 5. What are the four broad groups of genes that are target to genetic damage? What is the function of these groups and how does genetic damage alter their function? [16] 6. Describe the process how metastasis arises from the primary tumor.

[16]

7. Write short notes on: (a) Screening and treatment of infections. (b) Samples for analysis-cell lines, tissues, and biological fluids.

[8+8]

8. Write short notes on: (a) Possible outcomes of chemotherapy in cancer. (b) Possible outcomes of hormonal therapy in cancer. ⋆⋆⋆⋆⋆

1 of 1

[8+8]

Set No. 3

Code No: R05412306

IV B.Tech I Semester Supplimentary Examinations, March 2009 CANCER BIOLOGY (Bio-Technology) Time: 3 hours Max Marks: 80 Answer any FIVE Questions All Questions carry equal marks ⋆⋆⋆⋆⋆ 1. What is cell cycle and explain the process involved in the regulation of cell cycle? [16] 2. Discuss: (a) RB, retinoiblastoma (b) p53.

[8+8]

3. Explain: (a) Diol epoxides (b) DNA-PHA adduct.

[8+8]

4. Discuss the radiation dose and radiation risk ivolved in human cancers.

[16]

5. Write short notes on: (a) Pocket proteins. (b) Mechanism of p53 dominant negative mutations.

[8+8]

6. Write short notes on : (a) Role of matrix metallo proteins in remodeling of the extracellualar matrix. (b) Role of proteases in tumor cell invasion.

[8+8]

7. Write short notes on: (a) Safety and acceptability of screening tests of cancer. (b) Evaluation of cancer screening.

[8+8]

8. Write short notes on: (a) Recombinant anticancer drugs. (b) Problems of antibody therapy in cancer. ⋆⋆⋆⋆⋆

1 of 1

[8+8]

Set No. 4

Code No: R05412306

IV B.Tech I Semester Supplimentary Examinations, March 2009 CANCER BIOLOGY (Bio-Technology) Time: 3 hours Max Marks: 80 Answer any FIVE Questions All Questions carry equal marks ⋆⋆⋆⋆⋆ 1. What are different signal transduction proteins and explain their prognostic importance in cancer? [16] 2. How tumour cells can be formed? Explain the dissemination tumour cells by different routes? [16] 3. Discuss the metabolic activation of polycyclic aromatic hydrocarbons?

[16]

4. What is signal transduction? Explain the effects on cell-cell signaling during the radiation carcinogenesis? [16] 5. Explain : (a) Why p53 is regarded as “guardian of the genome”. (b) Process of cell cycle regulation by retinoblastoma.

[8+8]

6. Explain: (a) Relationship of cancer metastasis to normal tissue invasion events. (b) Role of lytic enzymes in the metastasis cascade.

[8+8]

7. Write short notes on screening of : (a) Cervix cancer. (b) Tumor antigens.

[8+8]

8. What are the new approaches to radiation therapy in relation to cancer? ⋆⋆⋆⋆⋆

1 of 1

[16]

Set No. 1

Code No: R05412307

IV B.Tech I Semester Supplimentary Examinations, March 2009 STRUCTURAL BIOLOGY (Bio-Technology) Time: 3 hours Max Marks: 80 Answer any FIVE Questions All Questions carry equal marks ⋆⋆⋆⋆⋆ 1. Explain the intracellular arrangement of membrane in phototropic bacteria. [16] 2. (a) Give a brief account of mitochondrial enzymes. (b) Explain clearly the “one gene-one enzyme” concept.

[6+10]

3. (a) What is cooperative transition? (b) Is protein folding a highly cooperative process, explain.

[6+10]

4. (a) What is sigler’s hypothesis? (b) Explain the sigler’s hypothesis for the nature of most indirect interactions between the repressor and operator? [4+12] 5. (a) “Covalent and Non covalent bonds are important for the structure and stability of biological molecules” Explain. (b) Explain the zwitter ionic structure of an amino acid.

[8+8]

6. Explain the following: (a) The evidences for the existence of E-S complex. (b) Multiple intermediates in steady state kinetics.

[8+8]

7. Write a note on: (a) Neutron diffraction. (b) Ramachandran plot.

[8+8]

8. (a) How will you determine the crystalline dimension by broadening of diffraction. (b) Compare and contrast the information obtained from NMR and X Ray Crystallographic Approaches to investigate macromolecular structure. [8+8] ⋆⋆⋆⋆⋆

1 of 1

Set No. 2

Code No: R05412307

IV B.Tech I Semester Supplimentary Examinations, March 2009 STRUCTURAL BIOLOGY (Bio-Technology) Time: 3 hours Max Marks: 80 Answer any FIVE Questions All Questions carry equal marks ⋆⋆⋆⋆⋆ 1. (a) What are poly saccharides. Explain how poly saccharide structure depends on various kinds of glucosidic bonds involved. (b) What are glucosidic bonds explain them with suitable examples.

[10+6]

2. (a) What are peptide bonds? How they are formed. Illustrate. (b) Explain how peptide bonds are kinetically quiet stable.

[4+12]

3. (a) Illustrate the functional importance of proteins. (b) Write the importance of multimeric DNA binding proteins.

[8+8]

4. (a) Illustrate the gel mobility shift technique to quantify the nucleic acid-protein interactions. (b) Explain how important are protein-protein interactions in eukaryotic transcription? [8+8] 5. Discuss the salient features of Ramachandran plot in terms of the main chain torsion angles of a dipeptide unit. Comment on the utility of this plot. [16] 6. Derive equation that describes the mechanism of enzyme action used as a model by Michelis & Menten. List the important assumptions used by Michelis & Menten to describe a rate equation for this reaction. [16] 7. Write short notes on: (a) Valence - bond theory. (b) Molecular orbital theory and its applications.

[6+10]

8. (a) Compare x-ray crystallography with NMR spectroscopy. (b) Explain neutron diffraction.

[8+8] ⋆⋆⋆⋆⋆

1 of 1

Set No. 3

Code No: R05412307

IV B.Tech I Semester Supplimentary Examinations, March 2009 STRUCTURAL BIOLOGY (Bio-Technology) Time: 3 hours Max Marks: 80 Answer any FIVE Questions All Questions carry equal marks ⋆⋆⋆⋆⋆ 1. Discuss the following conventions of ribonuclease. (a) Ball-and - stick modal. (b) Spiral-and-ribbon modal.

[8+8]

2. (a) Explain the importance of functional assays in isolating cloned genes. (b) How can you predict the glycophorin protein by membrane - spanning helix technique? [8+8] 3. Explain the following: (a) Quaternary structure of protein. (b) Edmens reagent. (c) Glutathione. (d) Zwitter ions.

[4×4]

4. (a) Explain the striking difference between DNA-binding proteins in prokaryotes and eukaryotes. (b) Explain how PCR technique used in study of molecular interactions.

[8+8]

5. Write short notes on following: (a) Intra-molecular interactions and hydrogen bonds in a double helix. (b) Formation of double helix.

[8+8]

6. (a) Distinguish between steady- state approximation and a steady state condition. (b) How would you establish the formal kinetic mechanism of a two substrate enzyme? [8+8] 7. (a) What are some of the factors that can lead to deviations in Beer’s Law and explain why? (b) How can non absorbing analytes be determined photometrically and what are some of the requirements for successful application? [8+8] 8. Explain the following: (a) General principles of NMR.

1 of 2

Set No. 3

Code No: R05412307

(b) Protein structure determination by Nuclear Magnetic Resonance Spectroscopy. [8+8] ⋆⋆⋆⋆⋆

2 of 2

Set No. 4

Code No: R05412307

IV B.Tech I Semester Supplimentary Examinations, March 2009 STRUCTURAL BIOLOGY (Bio-Technology) Time: 3 hours Max Marks: 80 Answer any FIVE Questions All Questions carry equal marks ⋆⋆⋆⋆⋆ 1. Differentiate clearly the following with suitable examples: (a) Endocrine hormones. (b) Exocrine hormones.

[8+8]

2. Give details about the calcium-calmodulin complex. Discuss the intra cellular effects of this complex. [6+10] 3. (a) Illustrate the functional importance of proteins. (b) Write the importance of multimeric DNA binding proteins.

[8+8]

4. (a) Explain the striking difference between DNA-binding proteins in prokaryotes and eukaryotes. (b) Explain how PCR technique used in study of molecular interactions. [8+8] 5. Explain the following: (a) Binding sites in protein. (b) Ribose puckering and Tertiary structure of tRNA.

[8+8]

6. Write short notes on: (a) (b) (c) (d)

Positive cooperativity Negative cooperativity homotropic cooperativity heterotrophic cooperativity.

[4×4]

7. Calculate the energy per photon and energy per mole of photons for radiation Of wave length: (a) (b) (c) (d)

600 550 400 150

nm nm nm pm

(red) (yellow) (blue) (X-ray).

[4×4]

8. Explain the following: (a) The origin of absorption edge. (b) Protein structure determination using NMR. ⋆⋆⋆⋆⋆ 1 of 1

[8+8]

Related Documents